RS Flashcards

1
Q

A specialty trainee from the medical ward shows you a CXR of a breathless patient. You observe splaying of the carina and a ‘double right heart border’. What is the most likely underlying diagnosis? [Book 1 Question 35]
A. Mitral stenosis.
B. Aortic stenosis.
C. Tricuspid incompetence.
D. Left ventricular aneurysm.
E. Coarctation of the aorta.

A

Mitral stenosis.
The findings describe left atrial enlargement, which is caused by mitral valve disease (stenosis or incompetence), ventricular septal defect (VSD), patent ductus arteriosus (PDA), atrial septal defect (ASD) with shunt reversal, and left atrial myxoma. Aortic stenosis produces left ventricular hypertrophy and eventually dilatation, the latter producing a prominent left heart border with inferior displacement of the cardiac apex. A left ventricular aneurysm produces a prominent bulge of the left heart border. Tricuspid incompetence produces an enlarged right atrium and thus a prominent right heart border on plain film. Coarctation produces left ventricular enlargement and inferior rib notching of the fourth to eighth ribs bilaterally if conventional and a ‘reverse figure 3’ sign: a prominent ascending aorta/arch and a small descending aorta, with an intervening notch.

How well did you know this?
1
Not at all
2
3
4
5
Perfectly
2
Q

A 45-year-old female patient with history of rheumatic fever as a child presents with progressive shortness of breath and paroxysmal nocturnal dyspnoea. Clinical examination reveals a pansystolic murmur associated with a mid-diastolic murmur with presystolic accentuation best heard over the cardiac apex. Clinical examination and plain film do not reveal evidence of heart failure, but several features of left atrial enlargement are noted. Which of the following is not one of those? [Book 2 Question 46]
a. Double atrial shadow on the right
b. Straightening of the right heart border
c. Elevation of the left main bronchus
d. Splaying of the carina
e. Displacement of the descending aorta to the left

A

Straightening of the right heart border
Left atrial enlargement results in straightening of the left heart border because of enlargement of the left atrial appendage. This is especially a feature of rheumatic mitral valve disease.

How well did you know this?
1
Not at all
2
3
4
5
Perfectly
3
Q

A junior doctor requests your opinion on a postero-anterior (PA) CXR of a 21-year-old man admitted with chest pain. She suspects that the patient has right middle lobe consolidation. What feature on the patient’s radiograph allows you to reassure her that the imaging appearances are secondary to pectus excavatum? [Book 1 Question 61]
A. Rightward displacement of the heart.
B. ‘Sevens’ appearance to ribs.
C. Indistinct right heart border.
D. Bilateral hilar enlargement.
E. Steeply angulated posterior ribs.

A

Sevens’ appearance to ribs.
Pectus excavatum is a relatively common thoracic skeletal anomaly. The majority of cases are isolated, although it is associated with Marfan’s syndrome and congenital heart disease. The majority of patients are asymptomatic. On the PA CXR, the heart is shifted to the left. The right heart border is indistinct (suggesting right middle lobe consolidation). The posterior ribs appear horizontal, and the anterior ribs are angulated steeply, giving rise to the ‘sevens’ appearance.

How well did you know this?
1
Not at all
2
3
4
5
Perfectly
4
Q

An 18-year-old woman with Poland syndrome is being assessed by plastic surgery for reconstruction. As part of her pre-operative work-up a CT chest is requested. What is the classic finding in this disorder? [Book 1 Question 56]
A. Absence of the sternal head of pectoralis major.
B. Hypoplastic clavicles.
C. Anterior protrusion of the ribs.
D. Bilateral breast aplasia.
E. Anterior protrusion of the sternum.

A

Absence of the sternal head of pectoralis major.
Poland syndrome is an uncommon congenital unilateral chest wall deformity characterized by partial or total absence of the greater pectoral muscle and ipsilateral syndactyly. Associated anomalies include ipsilateral breast aplasia and atrophy of the second to fifth ribs. Hypoplastic clavicles are a feature of cleidocranial dysostosis. Anterior protrusion of the ribs gives rise to pectus excavatum, whereas anterior protrusion of the sternum is seen in pectus carinatum.

How well did you know this?
1
Not at all
2
3
4
5
Perfectly
5
Q

A 45-year-old man presents with a history of cough and occasional haemoptysis. Plain chest radiograph demonstrates a right para-cardiac shadow with loss of the right heart border. Bronchoscopy demonstrates an endoluminal obstructive mass. The most likely site of the lesion would be: [Book 2 Question 24]
a. Right upper lobe anterior segmental bronchus
b. Right lower lobe lateral basal segmental bronchus
c. Bronchus intermedius
d. Right upper lobe posterior segmental bronchus
e. Right middle lobe bronchus

A

Right middle lobe bronchus
The features described are of an endoluminal lesion causing right middle lobe collapse. A lesion in the bronchus intermedius is likely to cause both middle and lower lobe collapse.

How well did you know this?
1
Not at all
2
3
4
5
Perfectly
6
Q

A 40-year-old has a routine chest radiograph as a part of pre-immigration work up. This demonstrates a mass on the left with loss of the upper left heart border. The descending aorta can, however, be seen despite the mass. Which of the following is the most likely location of the mass? [Book 2 Question 3]

a. Apico-posterior segment
b. Lingula
c. Anterior segment of the upper lobe
d. Posterior basal segment of the lower lobe
e. Lateral basal segment of the lower lobe

A

Lingula

This is an example of the silhouette sign where an anteriorly located lingular mass results in loss of the upper left heart border but preservation of the outline of the posterior descending aorta.

How well did you know this?
1
Not at all
2
3
4
5
Perfectly
7
Q

A 52-year-old male presents with dyspnoea and cough. A chest radiograph shows an ill-defined opacity in the right mid-zone, obscuring the heart border. A lateral view shows a thin wedge-shaped opacity with base in contact with the pleura antero-inferiorly and pointing postero-superiorly. What is the most likely diagnosis? [Book 4 Question 47]

a. right middle lobe collapse
b. right middle lobe consolidation
c. right lower lobe collapse
d. right lower lobe consolidation
e. encysted pleural fluid

A

right middle lobe collapse

In right middle lobe collapse, the horizontal fissure and lower half of the oblique fissure converge. This creates a wedge-shaped opacity on the lateral chest radiograph. On the frontal chest radiograph, there is an ill-defined mid-zone opacity. With right middle lobe consolidation, there is a mid-zone opacity with a well-defined superior margin, as the horizontal fissure remains in a normal position and is tangential to the radiograph beam. Both obscure the right heart border. Lower lobe collapse and consolidation cause basal opacity with loss of clarity of the right hemidiaphragm. The lateral view shows a triangular opacity at the right base posteriorly, larger in consolidation than collapse.

How well did you know this?
1
Not at all
2
3
4
5
Perfectly
8
Q

A patient is being investigated by his GP due to a history of dysphagia and occasional stridor. A CXR has been requested, which is reported as showing possible tracheal abnormality. A lateral CXR is requested, and this shows an abnormality in the retro-tracheal space (Raider triangle). Using your knowledge of the anatomy of this space and the diseases that may affect it, which of the following statements correctly describes an abnormality in this area and the effect it will have radiologically on the retro-tracheal space? [Book 1 Question 70]

A. A thickened tracheo-oesophageal stripe of 11mm will displace the trachea posteriorly.
B. An enlarged aorta bulges into the inferior aspect of the retro-tracheal space.
C. A subclavian artery aneurysm will be noted posterior to the tracheo-oesophageal stripe and will displace this anteriorly.
D. Mediastinal extension of a retropharyngeal abscess will widen the tracheo-oesophageal stripe superiorly.
E. A thyroid goitre extending retrosternally will displace the trachea posteriorly.

A

An enlarged aorta bulges into the inferior aspect of the retro-tracheal space.

Boundaries of the retro-tracheal space
* Anteriorly – posterior border of trachea
* Posteriorly – by the vertebrae
* Inferiorly – the aortic arch
* Posterior tracheal line – 2.5mm
* Tracheo-oesophageal line – 5.5 mm
* Extension of retro-pharyngeal abscesses usually occurs along the prevertebral space, posteriorly in the retro-tracheal space, thus not affecting the TOL.
* A normal retro-sternal goitre extending anterior to the trachea is not located in the retro-tracheal space.
* A normal subclavian artery is not present in the retro-tracheal space, but an aberrant left or right subclavian artery may be identified in the position described.

How well did you know this?
1
Not at all
2
3
4
5
Perfectly
9
Q

Into which structure does the thoracic duct normally drain? [Book 4 Question 6]

a. left brachiocephalic vein
b. left internal jugular vein
c. left subclavian vein
d. superior vena cava
e. junction of left subclavian and internal jugular veins

A

junction of left subclavian and internal jugular veins

The thoracic duct starts at the cisterna chyli at the level of T12. It passes behind the right diaphragmatic crus and crosses right to left in the thorax behind the oesophagus. It terminates by draining into the junction between the left subclavian and internal jugular veins, usually as two or three branches.

How well did you know this?
1
Not at all
2
3
4
5
Perfectly
10
Q

A 35-year-old man presents following a chest injury. A chest radiograph shows a smooth, curvilinear, tubular opacity adjacent to the right heart border. No other abnormality is seen. The accident and emergency team are requesting a CT of the chest. What is the most likely diagnosis? [Book 4 Question 57]

a. pulmonary contusion
b. pneumothorax
c. pericardial injury
d. extra-lobar sequestration
e. partial anomalous pulmonary venous return

A

partial anomalous pulmonary venous return

The appearances are classic of partial anomalous pulmonary venous return, which occurs in 0.3–0.5% of cases of congenital heart disease and is associated with atrial septal defects and hypogenetic lung. Contusions are seen as ill-defined opacities on CT, often with rib fractures. Anterior pneumothorax would cause increased conspicuity of the heart border. Pericardial injury produces a thick, irregular, shaggy, soft-tissue density adjacent to the heart border. Extralobar sequestration produces a triangular-shaped opacity adjacent to the diaphragm.

How well did you know this?
1
Not at all
2
3
4
5
Perfectly
11
Q

Which lung segments are separated by the superior accessory fissure? [Book 4 Question 75]

a. apical segment of lower lobes from other lower lobe segments
b. apical segment of right upper lobe from other upper lobe segments
c. superior segment of lingula from inferior segment of lingula
d. lingular segment of upper lobe from remainder of left upper lobe
e. right middle lobe from right lower lobe

A

apical segment of lower lobes from other lower lobe segments

The superior accessory fissure can be seen on both frontal and lateral radiographs. It is seen inferior to the horizontal fissure on the frontal projection and extends to the posterior chest wall on the lateral projection, whereas the horizontal fissure extends to the anterior chest wall. Other common accessory fissures are the inferior accessory fissure (between the medial basal segment of the lower lobe and other basal segments) and the azygos fissure (invagination of pleura into the upper lobe containing the azygos vein).

How well did you know this?
1
Not at all
2
3
4
5
Perfectly
12
Q

In normal anatomy, which vascular structure lies most anteriorly at the level of the thoracic inlet, posterior to the manubrium? [Book 4 Question 76]
a. left common carotid artery
b. brachiocephalic artery
c. superior vena cava
d. left brachiocephalic vein
e. right brachiocephalic vein

A

Left brachiocephalic vein
In the superior mediastinum the venous structures lie most anteriorly. The superior vena cava does not extend up to reach the thoracic inlet but is formed inferiorly by the convergence of the brachiocephalic veins. The right has a short vertical course to the right of the midline, while the left crosses from the root of the neck on the left to the right side of the superior mediastinum behind the manubrium, where it lies anterior to all of the other vascular structures.

How well did you know this?
1
Not at all
2
3
4
5
Perfectly
13
Q

In persistent left-sided superior vena cava, drainage usually occurs into which structure? [Book 4 Question 78]
a. left atrium
b. right atrium
c. normal right superior vena cava
d. hemiazygos vein
e. coronary sinus

A

Coronary Sinus
Persistent left-sided superior vena cava occurs in 0.3% of the general population and in 4.3–11% of patients with congenital heart disease. It is associated with atrial septal defects and azygos continuation of the inferior vena cava. It lies lateral to the aortic arch and anterior to the left hilum. It usually drains into the coronary sinus, but rarely drains into the left atrium, causing a left-to-right shunt. The normal right-sided superior vena cava is absent in 10–18% of cases of left-sided superior vena cava.

How well did you know this?
1
Not at all
2
3
4
5
Perfectly
14
Q

A CT scan performed on a patient shows a soft-tissue mass in the medial aspect of the left lung, invading the mediastinum between the aortic arch and pulmonary artery. Neither vessel is compromised. Which symptom may the patient have presented with? [Book 4 Question 79]
a. stridor
b. dysphagia
c. pain
d. swelling of face and neck
e. hoarse voice

A

Hoarse Voice
The space between the pulmonary artery and aortic arch is the aortopulmonary window, which contains the ligamentum arteriosum and the left recurrent laryngeal nerve. Invasion here by tumours can lead to paralysis of the left vocal fold, which attains a fixed adducted position, by involvement of the recurrent laryngeal nerve. Stridor and dysphagia could result from deeper invasion into the mediastinum, as the trachea and oesophagus form the medial border of the aortopulmonary window. Swelling of the face, neck and upper limbs occurs with superior vena cava obstruction, which is a feature of right sided mediastinal disease.

How well did you know this?
1
Not at all
2
3
4
5
Perfectly
15
Q

In normal anatomy, which structure lies immediately anterior to the left main bronchus at the left hilum? [Book 4 Question 84]
a. left pulmonary artery
b. left inferior pulmonary vein
c. left superior pulmonary vein
d. left phrenic nerve
e. left vagus nerve

A

Left superior pulmonary vein
The left pulmonary artery crosses over the superior aspect of the left main bronchus giving off the upper lobe artery and the inferior pulmonary artery, and then lies posterior to the left main bronchus. The left inferior pulmonary vein drains into the left atrium and does not reach the level of the left main bronchus. The vagus nerve lies posterior to the hilum adjacent to the oesophagus. The phrenic nerve lies anterior to all of the left hilar structures on the pericardium.

How well did you know this?
1
Not at all
2
3
4
5
Perfectly
16
Q

In an adult patient, which structure, along with the right atrium and superior vena cava, forms the right mediastinal border? [Book 4 Question 86]
a. right brachiocephalic vein
b. inferior vena cava
c. right ventricle
d. trachea
e. brachiocephalic artery

A

Right brachiocephalic vein
In an adult, the right mediastinal border normally comprises the right brachiocephalic vein, the superior vena cava and the right atrium. In young patients the thymus may produce a characteristic sail-shaped opacity over the right mediastinal border. The right tracheal wall can be seen as the paratracheal stripe through the right brachiocephalic vein and superior vena cava. The right ventricle does not form any part of the cardiac silhouette on a frontal chest radiograph. The brachiocephalic artery lies medial to the right brachiocephalic vein and does not form any part of the mediastinal border.

How well did you know this?
1
Not at all
2
3
4
5
Perfectly
17
Q

In anatomy of the aortic arch, after the normal configuration of vessels (brachiocephalic, left common carotid and left subclavian arteries), what is the next most common configuration seen? [Book 4 Question 88]
a. left vertebral artery arising from the arch between left common carotid and subclavian arteries
b. common origin of the brachiocephalic artery and left common carotid artery
c. right subclavian arising distal to the left subclavian artery
d. common origin of left common carotid and left subclavian arteries
e. double arch with common carotid and subclavian arteries arising from each side

A

Common origin of the brachiocephalic artery and left common carotid artery
The so-called normal aortic arch anatomy is seen in only 65% of people. The next most common configuration is where the left common carotid artery arises with the brachiocephalic artery in a common origin, seen in 13%, followed by the left common carotid arising from the brachiocephalic artery (bovine origin), seen in 9%. The left vertebral artery arising direct from the arch is seen in 2.5%, and the aberrant right subclavian artery (option c) occurs in 0.5%.

How well did you know this?
1
Not at all
2
3
4
5
Perfectly
18
Q

In the left lower lobe of the lung, the bronchi to which segments share a common origin? [Book 4 Question 95]
a. posterior basal and lateral basal
b. lateral basal and anterior basal
c. anterior basal and medial basal
d. medial basal and posterior basal
e. apical and posterior basal

A

Anterior basal and medial basal
There are five segments to the lower lobes of both lungs, but, unlike on the right, the medial basal and anterior basal segmental bronchi on the left usually have a common origin. The medial basal segment is small due to the cardiac indentation.

How well did you know this?
1
Not at all
2
3
4
5
Perfectly
19
Q

Which of the following most suggests active disease in an adult male with TB? [Book 3 Question 31]
A. Mediastinal lymph nodes more than 1cm in short axis diameter
B. Right-sided paratracheal lymphadenopathy
C. A Ghon focus
D. Ranke complex
E. Enlarged lymph nodes with low attenuation centres

A

Enlarged lymph nodes with low attenuation centres

Enlarged nodes greater than 2cm often have low attenuation centres on CT due to necrotic change, and are highly suggestive of active disease

How well did you know this?
1
Not at all
2
3
4
5
Perfectly
20
Q

Which of the following most strongly indicates post-primary rather than primary TB? [Book 3 Question 35]
A. The absence of lymphadenopathy
B. Consolidation in the mid zones
C. Self-limiting course
D. Pleural effusion
E. Atelectasis

A

**The absence of lymphadenopathy **

Although there may be overlap of features of primary and post-primary TB, the distinguishing features of post-primary TB include predilection for the upper lobes, the absence of lymphadenopathy and cavitation.

How well did you know this?
1
Not at all
2
3
4
5
Perfectly
21
Q

A 28-year-old Asian male immigrant presents with low-grade fever, weight loss and productive cough. There is no history of immunosuppression. Which of the following CXR findings is most in keeping with post-primary TB? [Book 1, Question 26]
A. Unilateral hilar lymphadenopathy.
B. Cavitating parenchymal opacity.
C. Pleural effusion.
D. Multiple bilateral non-calcified nodules <3 mm diameter.
E. Right lower lobe atelectasis.

A

Cavitating parenchymal opacity.

Primary TB
1. Lymphadenopathy is the radiologic hallmark
2. Parenchymal involvement → Homogenous consolidation
3. Obstructive atelectasis from enlarged lymph nodes.
4. Pleural effusion

Post-primary disease
1. Apical parenchymal opacity associated with cavitation. Other manifestations of post-primary TB are
2. Ill-defined opacities and
3. Tuberculomas

Multiple non-calcified nodules <3mm in diameter are characteristic of military TB.

How well did you know this?
1
Not at all
2
3
4
5
Perfectly
22
Q

A 22-year-old asthmatic presents with recurrent wheeze and productive cough with expectoration of brown sputum. Plain chest radiograph demonstrates multiple pulmonary infiltrates. Which of the following appearances on HRCT would be the most appropriate for acute allergic bronchopulmonary aspergillosis? [Book 2 Question 29]
a. Finger-in-glove opacity
b. Thick-walled cavity
c. Pleural thickening with or without an effusion
d. Endobronchial mass with distal atelectasis
e. Tree-in-bud appearance

A

Finger-in-glove opacity

Acute ABPA is seen as homogeneous, tubular, finger-in-glove areas of increased opacity in a bronchial distribution, usually involving the upper lobes. These shadows are related to plugging of airways by hyphal masses with distal mucoid impaction and can migrate from one region to another on HRCT. Thick-walled cavities and pleural thickening are features of saprophytic aspergillosis. Endobronchial lesion with distal atelectasis is seen mainly in chronic necrotising aspergillosis, whilst tree-in-bud appearance is seen with bronchiolitis in airway invasive aspergillosis.

How well did you know this?
1
Not at all
2
3
4
5
Perfectly
23
Q

A 25-year-old man with a history of asthma presents with flu-like symptoms. He has peripheral blood eosinophilia and elevated serum IgE. Chest radiograph shows hyperinflation, lobar consolidation and 1-2 cm ring shadows around the hilum and upper lobes. The peripheral bronchi are normal. Which of the following is the most likely diagnosis? [Book 3 Question 1]
A. Noninvasive aspergillosis
B. Tuberculosis
C. Invasive aspergillosis
D. Hypersensitivity pneumonitis
E. Allergic bronchopulmonary aspergillosis (ABPA)

A

ABPA

In an asthmatic patient, ABPA is strongly suggested by the presence of randomly distributed, central, moderate to severe bronchiectasis predominantly involving the upper lungs, bronchial wall thickening and centrilobular nodules.

ABPA [Radiopaedia]

CT findings include:
1. fleeting pulmonary alveolar opacities: common
2. centrilobular nodules representing dilated and opacified bronchioles
3. bronchiectasis
a. central, upper lobe saccular bronchiectasis
b. mucoid impaction results in a bronchocoele (finger in glove sign), (Y, V or toothpaste-like configuration)
c. high attenuation mucus, possibly calcification in impacted mucus
d. bronchial wall thickening: common
4. may progress to pulmonary fibrosis, upper lobe predominant
5. may cavitate: 10%

How well did you know this?
1
Not at all
2
3
4
5
Perfectly
24
Q

A 50-year-old chronic alcoholic and smoker presents with chronic cough. CXR shows bilateral upper lobe consolidation with nodular opacities and cavitation. These changes are slowly progressive over serial x-rays. A bronchoscopy is arranged and washouts from the upper lobes are negative for mycobacterial infection. Aspergillus titres are positive. How is the disease process best described? [Book 1 Question 66]
A. Allergic bronchopulmonary aspergillosis.
B. Bilateral aspergillomas with background COPD.
C. Semi-invasive aspergillosis.
D. Invasive aspergillosis.
E. Chronic aspiration pneumonia (aspergillus titres irrelevant).

A

Semi-invasive aspergillosis.

Aspergillus Lung Disease (Core Radiology Note)

  1. Allergic Bronchopulmonary Aspergillosis (ABPA)
    * Hypersensitive reaction to aspergillus
    * Commonly seen in patients with long-standing asthma
    * Recurrent wheeze, low-grade fever, cough, sputum with aspergillus hyphae.
    * CT – finger in glove sign (not specific)
    i. Upper lobe Bronchiectasis ii. Mucoid Impacting – high attenuation or even calcified.
  2. Aspergilloma
    * Conglomerate of aspergillus hyphae and cellular debris
    * In pre-existing cavity (Tb, Sarcoid)
    * Haemoptysis
    * CT – Monod Sign – crescent air surrounding mycetoma.
  3. Semi-invasive (chronic necrotising) Aspergillosis
    * Necrotising granulomatous inflammation to chronic aspergillus infection
    * Seen in debilitated, DM, alcoholic, COPD patients
    * Cough, chronic fever, haemoptysis is uncommon.
    * CT – segmental areas of consolidation, cavitation, and pleural effusions over months to years.
  4. Airway-invasive Aspergillosis
    * Deep to airway epithelial cells
    * Seen in immunocompromised patients.
    * Bronchiolitis to Bronchopneumonia.
    * CT – centrilobular and tree-in-bud nodules.
  5. Angio-invasive Aspergillosis
    * Invasion of arterioles
    * Seen in severely immunocompromised patients.
    * CT
    i. Halo sign – consolidation surrounded by GGO of haemorrhagic infarct
    ii. Air Crescent Sign – good prognostic sign.
How well did you know this?
1
Not at all
2
3
4
5
Perfectly
25
Q

A 64-year-old man with a history of alcoholism presents with acute onset fever and productive cough. What feature on his admission CXR would be in keeping with Klebsiella pneumonia as opposed to pneumococcal pneumonia? [Book 1 Question 36]
A. Lobar consolidation.
B. Parapneumonic effusion.
C. Reticulonodular opacity.
D. Bulging interlobar fissure.
E. Spherical opacity.

A

**Bulging interlobar fissure **
Klebsiella (Gram-negative) pneumonia occurs predominantly in older alcoholic men and debilitated hospitalized patients. On the CXR it appears as a lobar opacification with air bronchograms. A bulging interlobar fissure is secondary to inflammatory exudate, increasing the volume of the involved lobe. This sign, however, is not specific and is also seen with Haemophilus influenzae and Staphylococcus aureus. Pneumococcal (Streptococcus) pneumonia typically presents as lobar consolidation. Parapneumonic effusions are seen in up to 50%. Reticulonodular opacity is a recognized atypical presentation. In children it typically presents as a spherical opacity (round pneumonia).

How well did you know this?
1
Not at all
2
3
4
5
Perfectly
26
Q

**

A 35-year-old woman presents with chest infection and pyrexia and the plain film reveals dense lobar consolidation with bulging fissures. The likely micro-organism is: [Book 2 Question 2]
a. Legionella pneumophila
b. Pneumocystis carinii
c. Staphylococcus
d. Streptococcus
e. Klebsiella

A

Klebsiella
Klebsiella causes a dense pneumonia with bulging of fissures often associated with an empyema. Pneumococcal pneumonitis can also mimic this.

How well did you know this?
1
Not at all
2
3
4
5
Perfectly
27
Q

An 87-year-old male presents with fever and cough. A chest radiograph shows dense consolidation in the right mid-zone, which is seen to be in the apical segment of the lower lobe on a lateral view. The oblique fissure is seen to be bulging away from the consolidation. There is an associated effusion. What is the most likely diagnosis? [Book 4 Question 12]
a. Haemophilus influenzae pneumonia
b. Staphylococcus aureus pneumonia
c. Streptococcus pneumoniae pneumonia
d. Klebsiella pneumonia
e. Legionnaires’ disease

A

Klebsiella pneumonia
Klebsiella pneumonia is typically seen in elderly, debilitated men and produces dense, lobar consolidation, with bulging of the fissure sometimes seen. This may also be seen with pneumococcal pneumonia, but less commonly. Klebsiella may also cause empyema (the commonest cause) and patchy bronchopneumonia. Cavitation occurs in 50%. Staphylococcus aureus pneumonia has a broncho-pneumonic pattern which may coalesce, and cavitation is common. Multiple abscesses tend to occur in intravenous drug addicts. Effusions and empyema are common. Streptococcal pneumonia occurs in all ages, especially young adults, and often produces a lobar consolidation in the basal region. Haemophilus influenzae pneumonia has no characteristic appearance. Legionnaires’ disease causes a spreading consolidation of broncho-pneumonic type.

How well did you know this?
1
Not at all
2
3
4
5
Perfectly
28
Q

A 33-year-old male patient suffering from AIDS presents with constitutional symptoms and dry cough. His CD4 count is 150. HRCT is least likely to show: [Book 2 Question 33]
a. Pleural effusion
b. Ground glass changes
c. Bilateral interstitial infiltrates
d. Diffuse alveolar infiltrates
e. Pneumatocoeles

A

Pleural effusion
Pneumocystis carinii is the most common cause of pneumonia at this stage of the disease. Pleural effusions and lymphadenopathy are not features of PCP.

How well did you know this?
1
Not at all
2
3
4
5
Perfectly
29
Q

A 78-year-old male smoker with hospital admission 2 months ago for a dense left Middle Cerebral Artery (MCA) cerebral infarct presents. He has a 3-week history of low-grade fevers and weight loss. Chest radiograph reveals new left lower lobe consolidation with areas of cavitation and air fluid levels, but no hilar lymphadenopathy. Which is the most likely diagnosis? [Book 3, Question 21]
A. Hospital acquired pneumonia with lung abscess
B. Bronchogenic carcinoma
C. Bronchoalveolar cell carcinoma
D. Aspiration with anaerobic pneumonia
E. Traumatic contusion with lung cysts

A

Aspiration with anaerobic pneumonia
The superior segments of the lower lobes and posterior segments of the upper lobes are most affected in aspiration pneumonia. Thick-walled cavitation, frequently with air-fluid levels, can develop within weeks, mimicking post-primary TB.

How well did you know this?
1
Not at all
2
3
4
5
Perfectly
30
Q

Which of the following is the most common feature of Pneumocystis jiroveci infection on the CT of a 50-year-old man, 4 months post-bone marrow transplant? [Book 3 Question 36]
A. Diffuse bilateral ground-glass opacification
B. Pneumatoceles
C. Focal areas of consolidation
D. Pleural effusions
E. Mediastinal lymphadenopathy

A

Diffuse bilateral ground-glass opacification
There are often diffuse bilateral ground-glass opacities and less often, focal areas of consolidation, pleural effusions, and mediastinal lymphadenopathy. Approximately 1 / 3 of patients develop pneumatocoeles, usually in the upper lobes, which usually resolve spontaneously, although infrequently can cause a pneumothorax.

How well did you know this?
1
Not at all
2
3
4
5
Perfectly
31
Q

Which is the most common cause for tree-in-bud appearance on chest CT? [Book 3 Question 3]
A. Tumour emboli
B. Infection
C. Connective tissue disease
D. Aspiration pneumonitis
E. ABPA

A

Infection
Bacterial causes include mycobacterium tuberculosis, mycobacterium avium intra-cellulare complex, staph aureus. Viral causes include Cytomegalovirus (CMV) and respiratory syncytial virus. Fungal causes include invasive aspergillosis.

How well did you know this?
1
Not at all
2
3
4
5
Perfectly
32
Q

A 28-year-old HIV-positive IV drug user presents with progressive exertional dyspnoea, fever, and non-productive cough. CXR demonstrates bilateral para-hilar fine reticular opacities. There is no appreciable lymphadenopathy. What is the most likely diagnosis? [Book 1 Question 41]
A. Mycobacterium avium-intracellulare.
B. Pneumocystis jirovecii (formerly P. carinii).
C. Toxoplasmosis.
D. Coccidioidomycosis.
E. Candidiasis.

A

Pneumocystis jirovecii (formerly P. carinii)
* Commonest AIDS-defining opportunistic infection.CD4 <400 cells/mm 3.
* CXR
o Initially – normal.
o Eventually – a fine para-hilar reticular or ground-glass pattern.
o Pleural effusions and lymphadenopathy are uncommon.
Mycobacterium avium-intracellulare primarily affects the GI tract, but chest involvement in disseminated disease typically manifests as lymphadenopathy.
Diffuse reticular opacities and hilar lymphadenopathy are a feature of toxoplasmosis. Diffuse miliary nodules are seen in coccidioidomycosis.
Candida pneumonia demonstrates diffuse, bilateral nonsegmental airspace opacities.

How well did you know this?
1
Not at all
2
3
4
5
Perfectly
33
Q

A 24-year-old serviceman presents with insidious onset of fever, headache and worsening non-productive cough. His white cell count and erythrocyte sedimentation rate (ESR) are elevated, and serum cold agglutination is positive. He had failed to improve with initial antibiotic therapy. HRCT of chest demonstrates areas of ground-glass opacity, air-space consolidation, centrilobular nodules and thickening of bronchovascular bundles. What is the most likely diagnosis? [Book 1 Question 46]
A. Chlamydia pneumonia.
B. Mycoplasma pneumonia.
C. Pneumococcal pneumonia.
D. Legionella pneumonia.
E. Staphylococcal pneumonia.

A

Mycoplasma pneumonia.
The given clinical history is classical of mycoplasma pneumonia, which usually affects younger adults in closed populations such as prisons or the military. It is one of the most common causes of community acquired pneumonia in otherwise healthy individuals. Serum cold agglutination is positive in up to 70%. On HRCT areas of ground-glass attenuation tend to be around areas of consolidation. Centrilobular nodules and peri-broncho vascular thickening are common associated findings.

How well did you know this?
1
Not at all
2
3
4
5
Perfectly
34
Q

A 28-year-old woman presents with fever, myalgia, and cough. Due to a current community outbreak, the clinical team suspect that she has H1N1 influenza (swine flu). Which finding on her admission CXR is most strongly predictive of an adverse outcome? [Book 1 Question 51]
A. Upper lobe consolidation.
B. Bilateral central opacity.
C. Multizonal peripheral opacity.
D. Air bronchogram.
E. Pleural effusion.

A

Multizonal peripheral opacity
The majority of H1N1 influenza cases have been mild, but the 2009 strain can cause severe illness, including in young previously healthy persons. Radiological findings in four or more lung zones distributed bilaterally and peripherally, are significantly more often seen on the CXR obtained at admission in patients with poor outcome (requiring mechanical ventilation) compared to those with good clinical outcome. Central GGO is the most common radiographic abnormality but is not significantly associated with poor outcome. Pleural effusions are uncommon, although bilateral effusions are an independent predictor of short-term mortality in community acquired pneumonia. It should be noted that an initial normal CXR does predict against a poor outcome.

How well did you know this?
1
Not at all
2
3
4
5
Perfectly
35
Q

Which of the following features most favours melioidosis rather than tuberculosis? [Book 3 Question 24]
A. Lobar consolidation
B. Cavitation
C. Upper lobe predominance of nodular change
D. Rapid progression of clinical features
E. Lack of pleural involvement

A

Lack of pleural involvement
There is significant overlap between the features of melioidosis, pneumonia and TB. However, in melioidosis, simultaneous involvement of other organs is common. Pleural involvement such as effusion, empyema, mediastinal or hilar adenopathy are rare, making them useful differentiating features from TB.

Melioidosis (STATdx)

Infection caused by gram-negative bacillus Burkholderia pseudomallei

Acute melioidosis (85%)

Nodules – Small with irregular margins rapidly increase in size, coalesce, and may cavitate
Consolidation – Segmental, lobar, or multi-lobar ± cavitation
Pulmonary abscess

Chronic melioidosis (11%)

Mixed parenchymal opacities – Nodules, linear densities, and cavitation
Other manifestations – Lymphadenopathy, Pleural involvement is rare

Top Differential Diagnoses

Staphylococcus aureus pneumonia (acute melioidosis) – pneumatocele
Septic embolism (acute melioidosis) – evidence of distant infection
Tuberculosis (chronic melioidosis) – apical fibrosis, granuloma

How well did you know this?
1
Not at all
2
3
4
5
Perfectly
36
Q

A 35-year-old female presents with generalised malaise and cough, occasionally bringing up grape-skin-like material. Blood screen reveals eosinophilia. The patient has a history of travel to several countries worldwide. Which of the following plain film features is unlikely? [Book 2 Question 32]
a. Homogenous ovoid opacity
b. Cyst with a fluid level
c. Bilateral opacities
d. Calcification
e. Lower zone location

A

Calcification
The case describes hydatid disease. Hydatid cyst of the lungs can present as solid ovoid solitary or occasionally multiple lesions on plain films. When the cyst communicates with a bronchial tree, an air fluid level is demonstrated. Several other signs are described. Whilst bilaterality is less likely (up to 20%), calcification is extremely rare (0.7%)

Hydatid Pulmonary Cysts (STATdx)

Organism
Larval stage (metacestode) of cestodes (tapeworms) of genus Echinococcus (E. granulosus,
E. multilocularis, E. vogeli, E. oligarthrus)

Radiography

  • Solitary or multiple spherical or ovoid masses with well-defined borders
  • Variable size (1 cm to > 20 cm)
  • Tracheobronchial communication (rupture)
    o Meniscus or crescent sign: Cyst communication with bronchus (air between peri-
    cyst and exo-cyst)
    o Water lily or camalote sign: Undulating cyst membranes floating in fluid
  • Hydatid pneumonitis: Aspiration of hydatid material (vomica)
  • Pleural effusion; hydropneumothorax

CT

  • Spherical or ovoid fluid-filled cyst
  • Cyst wall enhancement
  • May involve mediastinum, heart, chest wall, pulmonary artery, diaphragm
How well did you know this?
1
Not at all
2
3
4
5
Perfectly
37
Q

Which of the following would be most in keeping with the appearance of pulmonary hydatid disease? [Book 4 Question 93]
a. multiple sub-centimetre nodules throughout both lungs with no cavitation
b. unilateral patchy alveolar changes with unilateral hilar lymphadenopathy
c. solitary ovoid mass with air–fluid level and floating debris
d. bilateral basal reticular change
e. 3 cm, rounded mass with central calcification

A

Solitary ovoid mass with air-fluid level and floating debris
The lungs are the second most frequent site affected by hydatid disease in adults, being involved in up to 25% of cases. Commonly, a solitary ovoid or spherical mass is seen in the lower lobes. These can be quite large, measuring up to 20cm in size. There is communication with the bronchial tree, producing air–fluid levels, and the wall of the cyst may be visible, and may appear as a curvilinear opacity or floating debris. Calcification may occur within the wall of the cyst in a few cases.

How well did you know this?
1
Not at all
2
3
4
5
Perfectly
38
Q

A 26-year-old undergoes a routine chest radiograph as part of the Australian residency application. The left upper lobe is hyperlucent and hyperexpanded and a lobular mass is demonstrated adjacent to the left hilum. CT reveals the presence of a dilated bronchus containing a plug of soft tissue. The surrounding lung is emphysematous. The most likely diagnosis is: [Book 2 Question 14]
a. Central carcinoid tumour
b. Bronchogenic cyst
c. Bronchial atresia
d. Cystic adenomatoid malformation
e. Congenital lobar emphysema

A

Bronchial atresia
* Congenital, usually discovered incidentally.
* Local obliteration of the proximal lumen of a segmental bronchus
* Most affected – apico-posterior segment of the left upper lobe.
* Distal airway beyond atresia
o Still produces mucous → mucoid impaction/mucocoele.
o Ventilated by collateral air shift → Hyperexpanded and lucent, oligaemic.
Congenital lobar emphysema can look similar, however there is usually no mucous plug and patients tend to present early.

How well did you know this?
1
Not at all
2
3
4
5
Perfectly
39
Q

A 19-year-old male presents following blunt chest trauma, with dyspnoea, chest pain and haemoptysis. A chest radiograph shows bilateral pneumothoraces, subcutaneous emphysema and bilateral fractures of multiple upper ribs. The pneumothoraces fail to resolve despite chest drains and he needs intubation and ventilation. Which finding at CT would be most suggestive of the diagnosis of tracheal injury? [Book 4 Question 30]
a. pneumomediastinum
b. bilateral pneumothorax
c. mediastinal haematoma
d. focal overdistension of endotracheal tube cuff
e. chylothorax

A

Focal overdistension of endotracheal tube cuff
Tracheobronchial injury secondary to blunt trauma presents with non-specific symptoms and signs. Persistent pneumomediastinum, pneumothorax or subcutaneous emphysema despite treatment is suggestive. Associated findings are fractures of the upper three ribs and posterior dislocation of the sternoclavicular joints. The diagnosis is confirmed by bronchoscopy. In intubated patients, focal overdistension of the cuff of the endotracheal tube is seen when the balloon bulges into the defect. Mediastinal haematoma is more suggestive of vascular injury and chylothorax is more suggestive of thoracic duct injury. These may occur in association with tracheobronchial injury.

How well did you know this?
1
Not at all
2
3
4
5
Perfectly
40
Q

A 20-year-old presents with shortness of breath and cough. A chest radiograph shows a well-defined right hilar mass. CT shows a 4 cm, rounded, soft-tissue mass arising from the mediastinum adjacent to the right side of the carina. The attenuation value of the lesion is 10 HU. No other abnormal findings are seen. What is the most likely diagnosis? [Book 4 Question 67]
a. bronchogenic cyst
b. pericardial cyst
c. carcinoid tumour
d. lymphoma
e. bronchogenic carcinoma

A

Bronchogenic cyst
Bronchogenic cyst is an abnormality of the ventral diverticulum of the primitive foregut and is the most common foregut abnormality in the thorax. Typical appearances are of a thin-walled cyst containing mucus or fluid. CT shows a well-defined mass of water density in 50% (0–10 HU) or slightly higher density in the rest (10–50 HU); 86% are mediastinal and 50% peri-carinal. Pericardial cysts largely occur in the cardio-phrenic angle and are rarely mediastinal. Like bronchogenic cysts, these may change shape with position and respiration. Carcinoids occur in the lungs within bronchi and are mainly central. These are solid and may calcify. Bronchogenic carcinomas are usually solid, and centrally located in 38% of cases, where they are usually small cell tumours. These occur in the older population. Lymphadenopathy secondary to lymphoma is usually solid (unless treated) and involves multiple nodes in 95% of cases. The anterior mediastinum and retrosternal lymph nodes are usually affected first.

How well did you know this?
1
Not at all
2
3
4
5
Perfectly
41
Q

A 73-year-old patient is involved in a road traffic accident (RTA) and sustains a head injury. He is intubated at the scene due to a low Glasgow Coma Scale (GCS). The patient is transferred for a CT chest as he is hypoxic. On reviewing the CT scan, you note widespread emphysema, consistent with the history of smoking. He has a narrowing of the trachea, immediately inferior to the distal margin of the endotracheal (ET) tube. This narrowing is caused by an endoluminal mass associated with a circumferential area of soft tissue that extends into the paratracheal space. There is no pneumomediastinum and no other lung injury is seen. What is the most likely cause? [Book 1, Question 5]
A. Post-tubation stenosis
B. Tracheal Papilloma
C. Non-small Cell Lung Cancer
D. Adenoid Cystic Carcinoma
E. Squamous Cell Carcinoma

A

Squamous cell carcinoma.
Tracheal malignancies make up 1–2% of all adult intrathoracic tumours and as such are uncommon. Malignant lesions make up 90% of all tracheal malignancies. Of these, squamous cell carcinomas are the most common, presenting in elderly patients with a history of smoking. Adenoid cystic carcinoma is the next most common, presenting in a younger age group and associated with a better prognosis. Benign lesions account for less than 10%. Non-small cell lung cancer (NSCLC) would be the leading differential diagnosis if this lesion was found endo-bronchially, but not in the trachea. NSCLC can cause tracheal narrowing, but as an extrinsic lesion. The history is too brief for post-intubation stenosis to be considered and this is not associated with a soft tissue mass.

How well did you know this?
1
Not at all
2
3
4
5
Perfectly
42
Q

A 60-year-old man presents with a 6-month history of shortness of breath, wheeze, and a recent episode of haemoptysis. Plain chest radiograph reveals partial right middle and lower lobe collapse. This is further confirmed on CT which also suggests an endobronchial lesion in the bronchus intermedius. It is FDG-avid on PET-CT scan. Bronchoscopy reveals a smooth submucosal lesion. The histopathology of the lesion is most likely to be: [Book 2 Question 51]
a. Mucoepidermoid carcinoma
b. Atypical carcinoid
c. Renal cell carcinoma metastasis
d. Adenoid cystic carcinoma
e. Squamous cell papilloma

A

Adenoid cystic carcinoma
Adenoid cystic carcinoma is the second most common malignancy of the central airways after squamous cell cancer and often presents as an endoluminal mass with an intact mucosa. Mucoepidermoid carcinoma is rare. Benign tumours are mostly of mesenchymal origin and are rare. Carcinoids in these locations are usually of the typical type.

How well did you know this?
1
Not at all
2
3
4
5
Perfectly
43
Q

A 34-year-old woman with a preceding history of chronic cough, weight loss and intermittent chest tightness presents with acute shortness of breath. CTPA reveals a large filling defect within the left pulmonary artery. Which radiological feature would most suggest a diagnosis of pulmonary artery sarcoma as opposed to pulmonary embolism? [Book 1, Question 21]
A. Mosaic lung perfusion.
B. Peripheral filling defect forming acute angle with arterial wall.
C. Peripheral filling defect forming obtuse angle with arterial wall.
D. Low attenuation filling defect occupying and expanding the entire luminal diameter.
E. Partial filling defect surrounded by areas of intravascular contrast enhancement.

A

Low attenuation filling defect occupying and expanding the whole luminal diameter.
Pulmonary artery sarcoma is a rare malignancy arising from the intima of the pulmonary artery. It is frequently misdiagnosed as PE, although there are features that help differentiation. Findings that favour pulmonary artery sarcoma include a low attenuation filling defect occupying the entire luminal diameter of the proximal or main pulmonary artery, expansion of the involved arteries and extraluminal tumour extension. A filling defect forming an acute angle with the arterial wall is seen in acute PE, whereas a filling defect forming an obtuse angle indicates organizing thrombus in chronic PE. A partial filling defect surrounded by areas of contrast enhancement is a feature of embolus floating freely within the lumen.

Low attenuation filling defect occupying and expanding the whole luminal diameter.

Pulmonary artery sarcoma is a rare malignancy arising from the intima of the pulmonary artery.
It is frequently misdiagnosed as PE, although there are features that help differentiation.
Findings that favour pulmonary artery sarcoma include a low attenuation filling defect
occupying the entire luminal diameter of the proximal or main pulmonary artery, expansion of
the involved arteries and extraluminal tumour extension. A filling defect forming an acute angle
with the arterial wall is seen in acute PE, whereas a filling defect forming an obtuse angle
indicates organizing thrombus in chronic PE. A partial filling defect surrounded by areas of
contrast enhancement is a feature of embolus floating freely within the lumen.

Pulmonary Artery Sarcoma (STATdx)

  • Large filling defect in PA refractory to anticoagulation
  • Radiography
    o Normal if intraluminal with no PA dilatation
    o PA dilatation: Hilar mass
    o Distal oligemia, pulmonary infarction, and extraluminal extension possible
  • CECT: Filling defect in PA; may exhibit contrast enhancement
  • MR: Distinction between tumour and thrombus
    o Tumour more likely to enhance
    o Acute thrombus and tumour thrombus may enhance
  • Cine MR for pulmonic valve assessment
  • FDG PET/CT: FDG uptake within tumour
How well did you know this?
1
Not at all
2
3
4
5
Perfectly
44
Q

You are carrying out a CT chest scan on a patient who is under the joint care of the respiratory physicians and the rheumatologists. The patient reports slowly progressing stridor. The patient has already been assessed by ear, nose, and throat (ENT) due to collapse of the nasal turbinates, but this is felt to be unconnected to his stridor. His inflammatory markers are elevated. A nasal biopsy showed an inflammatory infiltrate in the cartilage causing dissolution, but no granuloma formation or vasculitis. The CT shows smooth thickening of the anterior trachea, with early calcium deposition, with relative sparing of the posterior trachea. This pattern is most marked in the subglottic region. There is narrowing of the airway. This pattern is unaffected on the expiratory scan as compared to the inspiratory scan. The transverse diameter of the trachea is 60% of the sagittal diameter. What is the likely cause? [Book 1, Question 9]
A. Wegener’s granulomatosis.
B. Amyloidosis.
C. Relapsing polychondritis.
D. Mounier–Kuhn disease.
E. Tracheobronchomalacia.

A

Relapsing polychondritis.
Wegener’s granulomatosis and amyloidosis can both give a similar appearance to that described. Amyloid can occur as an isolated condition or as a part of systemic amyloidosis. It gives smooth narrowing, but can also give multifocal stenoses or plaques, and is frequently associated with calcification. Wegener’s commonly affects the subglottic region, giving an identical appearance, although it can cause a more irregular pattern of thickening and ulcer formation. Similarly, Wegener’s commonly affects the cartilage in the nose. However, histologically, Wegener’s causes vasculitis and granuloma formation. Mounier–Kuhn disease is also known as tracheobronchomegaly. It can be associated with tracheobronchomalacia, which may give a similar CT appearance to that described. The key difference is that this condition is characterized by a reduction in calibre of >50% of the airway lumen during expiration, as compared to inspiration. Relapsing polychondritis is a systemic condition also affecting the cartilage of the nose, ears, and joints.

How well did you know this?
1
Not at all
2
3
4
5
Perfectly
45
Q

Which of the following features is most likely to indicate malignancy in a solitary pulmonary nodule? [Book 3, Question 34]
A. Size > 3cm
B. Malignant growth rate
C. Spiculated margin
D. Age > 70
E. Smoking

A

C. Spiculated margin
In order of decreasing likelihood, the following are features suggestive of malignancy in an indeterminate pulmonary nodule:
* Spiculated margin
* Size > 3cm
* Age > 70 years
* Malignant growth rate
* Smoker
* Upper lobe location

How well did you know this?
1
Not at all
2
3
4
5
Perfectly
46
Q

Which of the following features is most likely to indicate a benign solitary pulmonary nodule? [Book 3, Question 38]
A. Age 30-39
B. Upper lobe location
C. Size < 1cm
D. Smooth margin
E. Age > 70

A

A. Age 30-39
Benign growth rate, benign calcification, age < 39 (answer) and a non-smoking history are the features most likely to favour a benign nodule.

How well did you know this?
1
Not at all
2
3
4
5
Perfectly
47
Q

A 53-year-old male smoker is under evaluation for a thoracic aortic aneurysm. Whilst reviewing pre- and post-contrast CT images of the chest, a 2.4cm lesion is seen in the left lower lobe, with a mural nodule. Which of the following features most favours a diagnosis of cavitating lung cancer rather than an intra-cavitary aspergilloma? [Book 3, Question 44]
A. Size of lesion
B. Contrast enhancement > 10HU
C. Wall thickness
D. Adjacent bronchiectasis
E. Volume loss in involved lobe

A

B. Contrast enhancement > 10HU
Degree of contrast enhancement is much higher in cavitating lung tumours. Adjacent bronchiectasis is more often seen in aspergillomas.

How well did you know this?
1
Not at all
2
3
4
5
Perfectly
48
Q

A 68-year-old miner develops an irregular opacity in the upper zone on plain chest radiograph. Which imaging feature would be more in favour of malignancy than progressive massive fibrosis (PMF)? [Book 2 Question 38]
a. Peripheral enhancement on contrast-enhanced MR
b. Peripheral location on axial images
c. Presence of calcification
d. High signal on T2-weighted images
e. Avid lesion on PET-CT

A

High signal on T2-weighted images
PMF has a peripheral location which moves towards the hilum on follow-up imaging. Calcification and cavitation may also be seen. PMF lesions can be FDG-avid on PET-CT. However, high signal in a mass on T2-weighted images is strongly suspicious for malignancy.

How well did you know this?
1
Not at all
2
3
4
5
Perfectly
49
Q

A 76-year-old female presents with haemoptysis and cough. A chest radiograph shows a mass in the right upper lobe that contains a crescent of air. Which feature on CT would make a cavitating malignancy more likely than aspergilloma? [Book 4 Question 13]
a. thin cavity wall
b. high-density central mass
c. enhancing central mass
d. calcification
e. adjacent bronchiectasis

A

Enhancing central mass
Aspergilloma appears as a solid mass in a thin-walled cavity, which lies in a dependent position and is mobile. A crescent-shaped airspace is typical. It may calcify and be associated with pleural thickening. In contrast, cavitating malignancy often has irregular thick-walled margins with a central mass that often enhances and is fixed and non-mobile. Calcification may occur in relation to malignancy but is usually adjacent to, rather than in, the lesion and is usually seen with a scar carcinoma. Neither causes high-density lesions. Adjacent bronchiectasis is more often seen when there has been previous infection such as tuberculosis, producing a cavity in which an aspergilloma develops.

How well did you know this?
1
Not at all
2
3
4
5
Perfectly
50
Q

Which of the following is the strongest feature to support a diagnosis of a hamartoma on investigation of a pulmonary mass? [Book 3 Question 12]
A. A smooth, lobulated contour
B. Chondroid calcification
C. Slow growth
D. The presence of an extra-adrenal paraganglioma
E. Cavitation

A

Chondroid calcification
Calcification occurs in approximately 20% of cases and is almost pathognomonic if of the chondroid ‘popcorn’ type.

How well did you know this?
1
Not at all
2
3
4
5
Perfectly
51
Q

A 50-year-old female is found to have a solitary pulmonary nodule on imaging. Which of the following features suggests that it is benign? [Book1 Question 43]
A. Irregular, spiculated margin.
B. Central ‘popcorn’ calcification.
C. Doubling time of 180 days.
D. Contrast enhancement of 25 Hounsfield units (HU).
E. SUV of 8 on PET-CT.

A

Central ’popcorn’ calcification
Irregular or spiculated margin, eccentric or stippled calcification, doubling time of 20–400 days, contrast enhancement of more than 15 HU, and high uptake (SUV > 2.5) on PET-CT are all features associated with a malignant lesion.

Diffuse, central nodular, and popcorn-like calcification, doubling time of more than 400 days, contrast enhancement of less than 15 HU, and low uptake (SUV < 2.5) on PET-CT are associated with benign lesions.

How well did you know this?
1
Not at all
2
3
4
5
Perfectly
52
Q

A 48-year-old male smoker with a pulmonary mass on CXR is investigated with CT. This shows a well-defined right upper lobe lesion measuring 3cm with attenuation of -156 Hounsfield Units (HU). Which is the most likely diagnosis? [Book 3 Question 26]
A. Tuberculoma
B. Metastatic from colorectal cancer
C. Histoplasmosis
D. Hamartoma
E. Pulmonary AVM

A

Hamartoma
Hamartomas are usually well-defined solitary lesions < 4cm. The presence of fat in the lesion is a diagnostic feature, although may not be present in a hamartomatous nodule in up to 1/3 of cases. The lesions may grow slowly over time.

How well did you know this?
1
Not at all
2
3
4
5
Perfectly
53
Q

A 56-year-old male presents with wheezing, cough, and recurrent chest infections. A chest radiograph shows right middle lobe consolidation. CT of the chest shows a 3 cm mass arising within the right middle lobe bronchus with distal collapse and consolidation. Which feature of the mass would make hamartoma more likely than carcinoid? [Book 4 Question 49]
a. central location
b. presence of calcification
c. cavitation
d. presence of fat
e. prominent enhancement

A

Presence of fat
Hamartomas are usually seen in the periphery of the lungs (two-thirds) with 10% being endobronchial. Calcification is seen in 15%, often popcorn type. Cavitation is rare but fat is seen in 50%. Carcinoids are usually located centrally and are endobronchial. Calcification is seen in one-third and they rarely cavitate. They do not contain fat and show prominent enhancement following contrast, as they are vascular.

Hamartomas: peripheral, Ca, rarely cavitate, fat +, don’t enhance. Carcinoid: central, Ca, rarely cavitate, no fat, enhance.

How well did you know this?
1
Not at all
2
3
4
5
Perfectly
54
Q

Which of the following descriptions would be most suggestive of a pulmonary hamartoma on imaging? [Book 4 Question 77]
a. round, 2 cm, soft-tissue mass with no calcification or fat, in a central location
b. irregular, 8 cm mass with cavitation and associated effusion
c. multiple lesions of 1–3 cm with calcification, throughout lungs
d. lobulated, 3 cm mass with calcification and fat, in a peripheral location
e. peripheral, 5 cm lesion with no calcification, and band-like opacity connecting it to the hilum

A

Lobulated, 3 cm mass with calcification and fat, in a peripheral location
Hamartomas are seen in 0.25% of the population and are the commonest benign lung tumour. Two-thirds are found peripherally. They are rarely multiple or cavitatory. Fifteen per cent calcify (classically popcorn) and 50% contain fat. Option (a) is more typical of carcinoid, while (c) is suggestive of multiple granulomas, probably secondary to chickenpox. Option (e) is characteristic of pulmonary arteriovenous malformation.

How well did you know this?
1
Not at all
2
3
4
5
Perfectly
55
Q

A 64-year-old smoker is referred by his GP for persisting consolidation which has failed to resolve despite multiple antibiotic therapies. Of note he has been apyrexic and inflammatory markers have not been particularly raised. The respiratory team request a CT of chest, which shows GGO and consolidation of almost the entire left lower lobe, delineated by the major fissure, which is not displaced. Air bronchograms are present, but there is no significant loss of volume or expansion of the lobe and no mediastinal or hilar adenopathy. No mass obstructing the left lower lobe bronchus (either endoluminal or extrinsic) is demonstrated and the bronchoscopy findings corroborate this (results from washings not yet available). A PET-CT is normal. What is the most likely pathology? [Book 1, Question 20]
A. Carcinoid tumour.
B. Bronchioloalveolar carcinoma.
C. Small cell carcinoma.
D. Tuberculosis (TB).
E. Klebsiella pneumonia.

A

Bronchioloalveolar carcinoma.
In this disease, the tumour spreads along the alveolar septa without invading alveolar walls. The air in the alveoli is replaced by tumour cells, producing consolidation and GGO. Mediastinal lymphadenopathy is rare; pleural effusion is common. Diagnosis is made by sputum/bronchial washing cytology or lung biopsy. It mimics other causes of air-space opacification such as pneumonia, haemorrhage, oedema etc. Disseminated adenocarcinoma, choriocarcinoma, or lymphoma might produce identical CT findings.
PET-CT is often negative in the case of both bronchioloalveolar carcinoma and carcinoid tumours of the lung.
Carcinoid is in the form of a focal mass, not diffuse consolidation.
Klebsiella pneumonia classically produces enlargement of the involved lobe, bulging of the fissures with the propensity for cavitation, and abscess formation.
Small cell carcinoma is usually positive on PET-CT.
TB is usually PET positive and has a predilection for the upper lobes, or apical segments of the lower lobes.

How well did you know this?
1
Not at all
2
3
4
5
Perfectly
56
Q

A 47-year-old male patient is referred to the respiratory physicians with a 1-year history of wheeze. He is a non-smoker. A CXR reveals subtle narrowing of the bronchus intermedius. A CT scan reveals a lesion with an endobronchial component, which narrows the airway significantly. The lesion also has an extraluminal component, which is 2 cm in diameter and has smooth margins. The lesion displays stippled calcification and no cavitation. Following contrast enhancement, the lesion enhances avidly in the arterial phase. You formulate a differential diagnosis based on these imaging features. The patient is not keen for intervention. Based on your suspicions, what would be the least invasive means of follow-up imaging to help achieve a diagnosis? [Book 1 Question 44]
A. PET-CT.
B. Bronchoscopy and biopsy.
C. Indium-111 octreotide single photon emission computed tomography (SPECT) CT.
D. MRI using T2WI and short tau inversion recovery (STIR) coronal imaging.
E. Bronchial angiography.

A

Indium-111 octreotide SPECT–CT.
Bronchial carcinoid
* Younger patients
* Not associated with smoking
* Central, hilar, perihilar, or endobronchial (unlike peripheral nodule of carcinoid tumour)
* Classical features
o a smooth nodule, narrowing or compressing the bronchus, or
o an endobronchial lesion
o Both have stippled calcification and demonstrate avid enhancement.
The degree of enhancement can simulate a pulmonary vascular malformation, which can be assessed with angiography, although MR angiography would be less invasive in this case. The definitive tissue diagnosis is commonly reached with bronchoscopy, but this can be associated with massive haemorrhage because of the vascular nature of this tumour. As such radiolabelled imaging, which is sensitive in 86% of patients, is a less invasive option.
FDG PET is often negative in cases of carcinoid because of the low metabolic activity of this tumour.
MRI characteristics of bronchial carcinoid have been well described but will be unlikely to significantly progress the diagnostic pathway in this case.

How well did you know this?
1
Not at all
2
3
4
5
Perfectly
57
Q

You are attending a lecture on lung cancer, but unfortunately you arrive late so you have missed the introduction. The lecturer is describing a subtype of lung cancer. The description is of a tumour that comprises 30% of all lung cancers. It typically occurs peripherally but can be central. This tumour can cavitate, but this occurs in only 4% of cases. Hilar and/or mediastinal involvement is seen in over half of cases on plain film radiography. What subtype of lung cancer is being described? [Book 1 Question 39]
A. Adenocarcinoma.
B. Bronchoalveolar carcinoma.
C. Squamous cell carcinoma.
D. Small cell carcinoma.
E. Giant cell carcinoma.

A

Adenocarcinoma.
These are all classical features of adenocarcinoma. Bronchoalveolar carcinoma is a subtype of adenocarcinoma. This comprises 2–10% of lung cancers. There are three subtypes: a solitary nodule (41%), multifocal nodules (36%), and peripheral consolidation (23%). Squamous cell carcinoma is only slightly less prevalent than adenocarcinoma. It cavitates in 86% of cases and typically occurs centrally. Small cell carcinoma comprises 18% of lung cancers. It usually presents on plain film radiography as hilar and/or mediastinal adenopathy. CT often detects lung opacities. Giant cell carcinoma is a poorly differentiated subtype of NSCLC that is capable of rapid growth and early metastasis.

How well did you know this?
1
Not at all
2
3
4
5
Perfectly
58
Q

You are taking the respiratory multidisciplinary team meeting. A respiratory physician has asked you to present two patients, both with incidentally detected solitary pulmonary nodules. Patient A is a 64-year-old male patient. He is a non-smoker. The lesion is 7 mm in diameter and smooth. Patient B is also a 64-year-old male, who smokes 30 cigarettes per day. His lesion is 5 mm in diameter. What follow-up would you recommend for these patients? [Book 1 Question 19]
A. Urgent positron emission tomography (PET) scan for both. Reassess with result.
B. CT within 6 months for Patient A. If unchanged repeat within 12 months. PET scan for Patient B and reassess with result.
C. CT scan within 12 months for Patient A. If unchanged further CT within a further 12 months. Serial 6 monthly CT scans for Patient B for 2 years.
D. CT scan within 12 months for both. If unchanged, both need a follow-up CT within a further 12 months.
E. Follow-up CT at 12 months for both. If unchanged, no further follow-up.

A

CT scan within 12 months for both. If unchanged, both need a follow-up CT within a further 12 months.

British Thoracic Society Guideline on Solitary Pulmonary Nodule
(2015)

Solid nodules

  1. Nodules with clear features of benign disease can be discharged
    a. Hamartoma
    b. Peri-fissural nodules
  2. Nodule size: <5 mm diameter or <80 mm3 volume → Discharge
  3. Nodule size: ≥5 to <6 mm diameter → CT at 1 year from baseline
    a. stable on basis of 2D diameter → CT 2 years from baseline with volume
    assessment
    b. stable on volumetry → Discharge
    c. volume doubling time >600 days → Discharge or CT surveillance up to patient
    d. volume doubling time 400-600 days → Consider biopsy or CT surveillance
    e. volume doubling time ≤400 days → Further workup or Definitive Mg
  4. Nodule size: ≥6 mm diameter or ≥80 mm3 volume → CT at 3 months from baseline
    a. volume doubling time ≤400 days → Further workup or Definitive Mgt
    b. volume doubling time >400 days → CT surveillance
  5. Nodule size: ≥8 mm diameter or ≥300 mm3 volume
    a. low risk of malignancy (<10% Brock) → CT surveillance
    b. higher risk of malignancy (≥10% Brock) → PET-CT and Herder model
    i. <10% risk of malignancy → CT surveillance
    ii. 10-70% risk of malignancy → Image-guided biopsy
    iii. >70% risk of malignancy → Excision or non-surgical management

Subsolid nodules

These include nodules with a partly solid or partly ground glass component.

  1. Nodule size: <5 mm diameter or <80 mm3 volume → discharge
  2. Nodule size: ≥5 mm diameter or ≥80 mm3 volume
    a. If there is previous imaging
    i. nodules stable for 4 years are discharged
    ii. nodules stable for less than 4 years undergo further surveillance and
    malignancy risk assessment
    b. if no previous imaging is available, CT in 3 months
    i. Resolved → Discharge
    ii. Stable
  3. low risk of malignancy (<10% Brock) → CT 1, 2, 4 years
  4. higher risk of malignancy (≥10% Brock) → D/W patients
    a. surveillance thin section CT at 1, 2 and 4 years
    b. image guided biopsy
    c. resection or non-surgical treatment
    iii. Growth/altered morphology → Resection or non-surgical
  5. offer repeat CT at 1, 2 and 4 years from baseline if the patient
    does not want resection/therapy
How well did you know this?
1
Not at all
2
3
4
5
Perfectly
59
Q

A 72-year-old female patient presents with a diagnosis lung malignancy obtained from bronchial washings. The CT shows a 4cm lesion in the right upper lobe with ipsilateral hilar and mediastinal lymphadenopathy in the 4R station. There is no chest wall invasion, and the lung lesion is surrounded by lung parenchyma. There is currently no evidence of infra-diaphragmatic disease. What is the TNM stage of this small cell lung cancer? [Book 1 Question 29]
A. T2a N1 M0.
B. T2b N1 M0.
C. T1b N2 M0.
D. T2a N2 M0.
E. None of these.

A

None of these.
[This answer does not apply to the most recent staging where small cell and non-small cell carcinoma are staged under the same system].
The TNM staging of lung cancer is not commonly used for staging small cell lung cancer. This cell type is particularly aggressive and often has occult metastases at the time of malignancy. The mainstay of treatment is with chemoradiotherapy, with imaging only used to stage disease as intrathoracic (limited or extensive) or extra-thoracic.

How well did you know this?
1
Not at all
2
3
4
5
Perfectly
60
Q

A 68-year-old patient has a CXR carried out due to a recurrent chest infection. The patient is a smoker. The CXR shows a solitary pulmonary nodule. A CT is carried out which demonstrates a 2.8-cm lesion in the right lower lobe as noted on CXR. This lesion is spiculated. There is a second lesion noted in the right lower lobe that is 1.2 cm in size and was not visible on the CXR. There is a 0.8-cm ipsilateral peri-bronchial lymph node identified. There are no evident metastases. A PET-CT is carried out which shows an SUV max of 8 in both pulmonary lesions. There is no uptake in the lymph node. No metastases are identified. A biopsy confirms non-small cell lung cancer. Based on the available imaging, what is the stage of this lesion? [Book 1 Question 34]
Stage 1A
Stage 1B
Stage 2B
Stage 2B
Stage 3A

A

**Stage 2B **
The TNM definitions, on which the staging system is based, were recently updated in 2010. In this update, the nodal classifications were not changed, but the T staging was updated, as was M staging. Sub-classifications were added to T1, with lesions <2 cm being T1a and lesions between 2 and 3 cm being T1b. Lesions between 3 and 5 cm are T2a and between 5 and 7 cm are T2b. Lesions over 7 cm are T3, as are synchronous lesions within the primary lobe, as in this case. Multiple lesions within the primary lobe were formerly T4. In this question, the lymph node is not enlarged by size criteria and is not FDG avid on PET. While this may yet still be involved, on imaging findings alone, this lesion should be classed as N0. The absence of metastases is obviously M0. Thus, this patient is T3 N0 M0, which corresponds to stage 2b. T2b N1 tumours are also in this stage. Stage 2a lesions are T2a N1 or T2b N0. Stage 3a lesions are T3 or less with N2 disease or T3 N1 lesions.

Lung Cancer Staging (8 th Edition – 2017)

T Staging

T1 < or = 3cm [Substage a, b, c – 1, 2, 3 cm]
T2a [3.1-4 cm] or main bronchus without carina
T2b [4.1-5 cm] or visceral pleura, hilar fat, or atelectasis
T3 [5-7 cm] or synchronous nodule in the same node***, parietal pleura/pericardium, chest
wall, phrenic nerve
T4 [>7 cm] or visceral pericardium, mediastinum, vertebra, diaphragm, carina
N Staging

N1 ipsilateral peripheral nodes up to hila
N2 ipsilateral mediastinal, carina
N3 any contralateral, or supraclavicular/scalene of any side

M Staging

M1a Intra-thoracic, Pleural/pericardial effusion
M1b Single organ extra-thoracic
M1c Multiple organ extra-thoracic

Overall

All M1c → Stage IVB
Other M’s → Stage IVA

N1 → Start from IIB until T3, then IIIA
N2 → Start from IIIA until T3, then IIIB
N3 → Start from IIIB until T3, then IIIC

T1 a, b, c → IA 1, 2, 3
T2 a, b → IB, IIA
T3 → IIB
T4 → IIIA

How well did you know this?
1
Not at all
2
3
4
5
Perfectly
61
Q

An 80-year-old man presents with haemoptysis and a mass on chest radiograph. A biopsy shows non-small-cell lung cancer. CT of chest shows a 4 cm, right middle lobe mass with pleural tethering but no chest wall invasion. Lymph nodes are seen at the right hilum (17 mm short axis), in the subcarinal space (20 mm short axis) and in the aortopulmonary space (8 mm short axis). No other abnormalities are seen. What is the TNM stage? [Book 4 Question 31]
a. T2 N1 M0
b. T2 N2 M0
c. T2 N3 M0
d. T3 N1 M0
e. T3 N2 M0

A

T2 N2 M0
The T stage is T2, as the lesion is over 3cm but there is no chest wall or mediastinal invasion or other associated feature. The nodes at the right hilum (N1) and in the subcarinal space (N2) are significantly enlarged, whereas the node in the aortopulmonary space (N3) is not (<10mm short axis), hence the N stage is N2.

How well did you know this?
1
Not at all
2
3
4
5
Perfectly
62
Q

A 55-year old man has a repeat chest radiograph which demonstrates a persistent patch of consolidation four months after a previous radiograph. Bronchioloalveolar carcinoma (BAC) is suspected. Which of the following makes the diagnosis less likely? [Book 2 Question 21]
a. Low attenuation consolidation
b. Negative PET-CT
c. Central location
d. long history of smoking
e. Associated cavitation

A

Central location
BAC can present in a local form as a mass, usually peripheral, subpleural in location or as diffuse persistent/progressive consolidation in patients with a history of smoking. The area of consolidation is often of low attenuation on CT due to copious mucin production. It is the second most common type of malignancy associated with cavitation.

How well did you know this?
1
Not at all
2
3
4
5
Perfectly
63
Q

The plain chest radiograph of a 52-year-old male presenting with cough and haemoptysis reveals a veil-like opacity over the left upper zone. CT reveals an endobronchial lesion in the left upper lobe bronchus causing lobar collapse. Bronchoscopic biopsy is least likely to reveal: [Book 2 Question 40]
a. Squamous cell carcinoma
b. Carcinoid
c. Lymphoma
d. Metastatic renal cell cancer
e. Bronchioloalveolar carcinoma

A

Bronchioloalveolar carcinoma
BAC usually presents as a peripheral, subpleural mass or persistent patch of consolidation. All the others can present as endobronchial lesions.

How well did you know this?
1
Not at all
2
3
4
5
Perfectly
64
Q

A 62-year-old man presents with right shoulder pain which radiates down his arm. A plain radiograph confirms the presence of a right apical mass with destruction of the surrounding ribs. CT-guided biopsy is performed and is likely to reveal: [Book 2 Question 22]
a. Large cell lung cancer
b. Squamous cell cancer
c. Small cell lung cancer
d. Adenocarcinoma
e. Carcinoid

A

Squamous cell cancer
The case describes a Pancoast tumour for which squamous is the most common cell type.

How well did you know this?
1
Not at all
2
3
4
5
Perfectly
65
Q

A 60-year-old man presents with progressive breathlessness. The plain chest radiograph reveals reticular shadowing in the right lower zone but is otherwise unremarkable. HRCT demonstrates the presence of beaded thickening of the interlobular septae forming a polygonal reticular network in the right lower lobe with central dots within. There is also a small pleural effusion. Which of the following is the most likely cause? [Book 2 Question 59]
a. Sarcoidosis
b. Bronchogenic carcinoma
c. Heart failure
d. Extrinsic allergic alveolitis
e. Fibrosing alveolitis

A

Bronchogenic carcinoma
The changes described indicate lymphangitis carcinomatosis. Whilst similar appearances are seen in sarcoidosis, the changes are usually in the upper lobe and pleural effusion is rare. In heart failure, the interlobar septal thickening is usually smooth and usually bilateral. In EAA, pleural effusion is rare, and changes are more broncho centric and bilateral. The polygonal structure is usually distorted in both EAA and cryptogenic fibrosis, where the changes are again subpleural.

How well did you know this?
1
Not at all
2
3
4
5
Perfectly
66
Q

A 47-year-old female Caucasian smoker presents with persistent cough and haemoptysis. CT shows a 2.2 cm soft tissue mass in the left lower lobe with coarse calcific foci and no mediastinal, hilar, or axillary lymphadenopathy. A small endobronchial lesion is also seen in the left lower lobe bronchus. What is the most likely diagnosis? [Book 3 Question 22]
A. Bronchogenic carcinoma
B. Pulmonary TB
C. Lymphoma
D. Fibrosing mediastinitis
E. Bronchial carcinoid

A

A. Bronchogenic carcinoma
A typical carcinoid is a small mass (< 2.5cm) with no associated lymphadenopathy. Calcifications are seen in up to 26% of cases on CT. The iceberg phenomenon is the presence of a small endobronchial component associated with a larger extra-bronchial component.

How well did you know this?
1
Not at all
2
3
4
5
Perfectly
67
Q

Which of the following indicates T3 rather than T4 lung cancer? [Book 3 Question 20]
A. Invasion of the oesophagus
B. Invasion of the trachea
C. Invasion of the pericardium
D. Malignant pleural effusion
E. Invasion of the vertebral body

A

Invasion of the pericardium
T3 disease features include a tumour of any size less than 2cm from the carina, invasion of the parietal pleura, chest wall, diaphragm, mediastinal pleura, pericardium, pleural effusion or satellite nodule in the same lobe.
T4 disease is characterised by invasion of the heart, great vessels, trachea, oesophagus, vertebral body, carina, or the presence of a malignant pleural effusion.

How well did you know this?
1
Not at all
2
3
4
5
Perfectly
68
Q

A 60-year-old female underwent a right pneumonectomy for bronchogenic carcinoma. Which feature on plain chest radiograph would become of worry seven days after surgery? [Book 2 Question 28]
a. A sequential increase in the fluid level
b. Shift of the previously central trachea to the right
c. Shift of the previously central trachea to the left
d. Elevation of the right hemi-diaphragm
e. Shift of the cardiac silhouette to the right

A

Shift of the previously central trachea to the left
All the other changes are expected changes at this stage following a pneumonectomy. However, contralateral shift of the trachea may be indicative of a post-surgical bronchopleural fistula.

How well did you know this?
1
Not at all
2
3
4
5
Perfectly
69
Q

A 48-year-old female with known lung cancer undergoes an 18 FDG PET/CT scan. The tumour is highly FDG avid. Several areas of moderate uptake are noted on the study. Which of the following findings is of concern about metastatic disease? [Book 4 Question 32]
a. diffuse thyroid uptake
b. symmetrical nasopharyngeal tonsil uptake
c. diffuse uptake in caecal wall
d. focal paravertebral muscle uptake
e. focal uterine cavity uptake

A

Focal paravertebral muscle uptake
FDG uptake occurs at many sites due to physiological uptake. Muscle uptake is variable, but usually occurs throughout a muscle when physiological. Focal muscle uptake is of concern, although benign causes are not uncommon. Diffuse thyroid uptake is usually physiological, but may indicate thyroiditis, whereas focal thyroid uptake may be malignant in 20–50% of cases. Tonsillar uptake if diffuse and symmetrical is likely to be benign, but asymmetrical or focal uptake requires further assessment. Caecal uptake, if diffuse, is probably due to lymphoid uptake, but, again, focal uptake must be assessed further. FDG uptake in the uterine cavity is usually due to menstruation.

How well did you know this?
1
Not at all
2
3
4
5
Perfectly
70
Q

A 78-year-old man presents with superior vena caval syndrome. A CT scan shows an irregular mass in the superior mediastinum causing near-total occlusion of the superior vena cava. There is no sign of respiratory compromise or raised intracranial pressure. Which of the following would be the most appropriate next step in the patient’s management? [Book 4 Question 35]

a. obtain tissue diagnosis
b. chemotherapy
c. radiotherapy
d. stenting of superior vena cava
e. surgical bypass

A

Obtain tissue diagnosis

Wherever possible, definitive diagnosis should be obtained, as it will enable the best possible treatment. Dependent upon the cause of the obstruction, the correct therapeutic option can then be employed. Radiotherapy often gives a good response, but, in small-cell lung cancer, chemotherapy often gives good results. Stenting should be used when radiotherapy or chemotherapy has failed. Surgical therapy is usually reserved for benign causes where conservative options have failed.

How well did you know this?
1
Not at all
2
3
4
5
Perfectly
71
Q

A 52-year-old male presents with mild dyspnoea. A chest radiograph shows a raised left hemidiaphragm which demonstrates paradoxical movement on fluoroscopy. Which of the following would be the most likely cause? [Book 4 Question 59]

a. Pancoast’s tumour
b. left lower lobe tumour
c. mediastinal small cell carcinoma
d. eventration
e. cerebrovascular accident

A

Mediastinal Small Cell Carcinoma

Diaphragmatic elevation with paradoxical motion is usually due to phrenic nerve paralysis. Mediastinal tumours are one of the commonest causes. Pancoast’s tumours invade the brachial plexus, though they can uncommonly invade the phrenic nerve. Eventration tends not to show paradoxical motion, though it can if large. Strokes may cause diaphragmatic elevation, but due to bilateral innervation of the diaphragm, this is not usually due to phrenic nerve involvement and has other causes, such as weakness of chest wall muscles.

How well did you know this?
1
Not at all
2
3
4
5
Perfectly
72
Q

A 44-year-old male presents with a solitary pulmonary nodule on a chest radiograph performed for a suspected chest infection. CT shows this to be 20 mm in diameter with a central cavity and smooth internal walls. No additional findings are seen. Which investigation should be arranged? [Book 4 Question 61]

a. none; findings are entirely benign
b. CT enhancement study
c. interval volumetric CT
d. contrast MRI
e. 18 FDG PET/CT scan

A

18 FDG PET/CT scan

Although the imaging features suggest a benign nodule, malignancy cannot be excluded, and further investigation is warranted. CT enhancement studies may be helpful in solid nodules but are of less value when a nodule is cavitating. An 18 FDG PET/CT scan has both high sensitivity and specificity in assessing solitary pulmonary nodules, with accuracy over 90%. There is a high specificity for a lesion that shows no FDG uptake being benign. Follow-up is still recommended to ensure no growth and exclude a very low-grade tumour. Interval CT may be considered an option, but in a young patient, unless there are exceptional circumstances, aiming to characterize the lesion is considered more appropriate. MRI is of no proven value in the evaluation of lung nodules.

How well did you know this?
1
Not at all
2
3
4
5
Perfectly
73
Q

A 56-year-old male has a cough. A chest radiograph and CT chest show a 2cm rounded mass in the apex of the left lung not amenable to biopsy. An 18FDG PET/CT scan is arranged for further assessment. Which technique may help to improve characterization of the lesion as benign or malignant using the standardized uptake value (SUV)? [Book 4 Question 66]

a. maximum SUV corrected for lean body mass
b. maximum SUV corrected for body weight
c. metabolic tumour burden (volume average SUV)
d. dual time point assessment of SUV
e. assessment of SUV centrally and peripherally in the lesion

A

Dual time point assessment of SUV

Dual time point assessment involves measuring the SUV at two time points to assess for change. In malignant lesions the SUV rises with time, whereas with benign lesions this tends to remain static. The maximum SUV corrected for lean body mass or weight is the standard measurement used for assessment of metabolic activity of lesions on PET. The metabolic tumour burden is currently not used in clinical practice. The assessment of uptake in the peripheral and central areas of a lesion is not a recognized technique.

How well did you know this?
1
Not at all
2
3
4
5
Perfectly
74
Q

A 64-year-old male presents with worsening shortness of breath and haemoptysis. A chest radiograph shows a right hilar mass with extensive reticulation in the ipsilateral lung, with Kerley A and B lines and reduced lung volumes. The left lung is clear. CT of the chest demonstrates the right hilar mass, and a thickened parenchymal polygonal network within the mid and lower zones of the ipsilateral lung. Beaded thickening of the interlobular septa is also noted. What is the most likely diagnosis? [Book 4 Question 69]

a. lymphangitis carcinomatosis
b. idiopathic pulmonary fibrosis
c. extrinsic allergic alveolitis
d. histiocytosis
e. sarcoidosis

A

Lymphangitis carcinomatosis

Lymphangitis carcinomatosis is tumour cell accumulation within connective tissue, causing lymphatic obstruction. This leads to interstitial oedema. The chest radiograph may be normal, but there is often reticulonodular change with Kerley A and B lines. An ipsilateral hilar or mediastinal mass may be seen. Pulmonary fibrosis has a peripheral predominance and tends to be bilateral. Extrinsic allergic alveolitis and histiocytosis tend to be bilateral. Sarcoidosis usually produces a more nodular pattern and there is predominance in the upper lobes. Polygonal structures on CT are uncommon in sarcoidosis.

How well did you know this?
1
Not at all
2
3
4
5
Perfectly
75
Q

In patients undergoing lung resection for malignancy, which imaging investigation is the best predictor of postoperative lung function? [Book 4 Question 71]

a. perfusion scintigraphy
b. ventilation scintigraphy
c. ventilation and perfusion scintigraphy
d. helical CT of the lungs
e. dynamic MRI of the lungs

A

It is possible to estimate postoperative lung function (FEV 1 (forced expiratory volume in 1 s)) with ventilation and perfusion scintigraphy, either in isolation or combination. However, ventilation scintigraphy is the best predictor of postoperative FEV1. This does underestimate the value; therefore, patients with borderline lung function should not necessarily be denied surgery because of the result. CT and MRI have not been shown to be helpful in assessing postoperative lung function.

How well did you know this?
1
Not at all
2
3
4
5
Perfectly
76
Q

Plain radiographs of a previously well 60-year-old man with lower limb pain show lamellar periosteal reaction in the dia-metaphyseal regions. Bone scintigraphy demonstrates symmetrical increased uptake along the cortical margins in the dia-metaphyseal regions. What is the most likely underlying thoracic cause? [Book 3 Question 29]

A. Pulmonary haemangioma
B. Pleural fibroma
C. TB
D. Blastomycosis
E. Malignant mesothelioma

A

Pleural fibroma

HPOA
* Malignant Causes
o Primary bronchogenic carcinoma especially NSCLC
o Pulmonary metastasis from extra-thoracic malignancies
* Benign Causes
o Pleural fibroma (answer)
o Bronchiectasis

How well did you know this?
1
Not at all
2
3
4
5
Perfectly
77
Q

A 65-year-old man presents with painful wrists and ankles. There is no digital clubbing. A chest radiograph shows a well-defined pleural mass, forming an obtuse angle with the chest wall. CT confirms an ovoid, pleurally based, enhancing mass with no bone destruction, effusion, or volume loss. Radiographs of the wrists and ankles show symmetrical periosteal reaction. What is the most likely diagnosis? [Book 4 Question 17]

a. hypertrophic osteoarthropathy with bronchogenic carcinoma
b. hypertrophic osteoarthropathy with malignant mesothelioma
c. hypertrophic osteoarthropathy with pleural fibroma
d. rheumatoid arthritis
e. tuberculosis

A

HPOA with pleural fibroma

The periosteal reaction is typical of hypertrophic osteoarthropathy (HOA), which has many causes, both intra- and extra-thoracic. It is associated with bronchogenic carcinoma and malignant mesothelioma, but features of the described lesion are not typical of either of these, and are more in keeping with pleural fibroma. These are associated with HOA in 20–35% of cases and rarely with clubbing. They may arise in pleura adjacent to the chest wall or in a fissure and can vary in size from 2 cm to 30cm in diameter. Tuberculosis is a rare cause of HOA. Rheumatoid arthritis is associated with bilateral periosteal reactions, but not HOA.

How well did you know this?
1
Not at all
2
3
4
5
Perfectly
78
Q

A 50-year-old builder undergoing CT of the chest has pleural thickening and calcification along the diaphragmatic pleura. Which is the most specific distinguishing feature to indicate malignant rather than benign pleural disease? [Book 3 Question 23]

A. Circumferential pleural thickening
B. Nodularity of the pleural thickening
C. Parietal pleural thickening greater than 1cm
D. Mediastinal pleural involvement
E. Pleural effusion

A

Nodularity of the pleural thickening

On multivariate analysis, the CT findings of rind-like pleural involvement, mediastinal pleural involvement and pleural thickness > 1 cm were independent findings in differentiating malignant pleural mesothelioma from other malignant pleural disease, with specificity values of 0.85, 0.67 and 0.82, respectively.

How well did you know this?
1
Not at all
2
3
4
5
Perfectly
79
Q

A 72-year-old former ship builder has presented with increasing shortness of breath to the respiratory physicians. A CXR reveals a pleural mass. You carry out a CT scan, which shows a 1cm diameter area of pleural thickening extending along the lateral chest wall inferiorly to the diaphragm. On coronal reconstructions the diaphragm appears smooth. There are several >1-cm nodes noted in the ipsilateral hilum as well as a solitary 1.2-cm node noted in the contralateral hilum. Following discussion with thoracic surgery a core biopsy is done, which confirms the diagnosis of malignant mesothelioma. An MRI is carried out. The lesion is increased signal on T2. The enlarged nodes are also identified. On post-gadolinium coronal fat saturation sequences a focus of high signal is noted to extend from the parietal pleura through the diaphragm to involve the peritoneum. A single focus of chest wall invasion is also noted. PET-CT shows high uptake in the lesion with a standard uptake value maximum (SUV max) of 25. All nodes except for the contralateral node demonstrate uptake. Which of these factors means this tumour is inoperable? [Book 1, Question 24]

A. The contralateral enlarged node noted on CT and MRI.
B. The tissue diagnosis of malignant mesothelioma.
C. The high SUV max.
D. The chest wall disease noted on MRI.
E. The diaphragmatic disease noted on MRI.

A

The diaphragmatic disease noted on MRI.

The finding of peritoneal disease upstages this tumour to Stage 4, which is inoperable. CT is the main imaging modality in malignant mesothelioma, which is sufficient in a lot of cases that are obviously inoperable on this modality alone. MRI has been shown to be superior to CT in the detection of local invasion, hence the normal CT appearance in this case, where peritoneal disease was noted on MRI. PET or PET-CT are better at detecting nodal disease than either CT or MRI. As such, in some cases all three modalities are necessary to stage disease.

How well did you know this?
1
Not at all
2
3
4
5
Perfectly
80
Q

A 76-year-old man presents with newly diagnosed malignant mesothelioma. Which of the following findings would preclude resection of mesothelioma? [Book 3 Question 5]

A. Enhancement of interlobular fissures on MRI
B. An area of chest wall invasion
C. Subdiaphragmatic extension
D. Stage T3 disease
E. Enlarged ipsilateral mediastinal nodes

A

Subdiaphragmatic Extension

Imaging findings of deep myocardial invasion, multifocal chest wall invasion or subdiaphragmatic extension are contraindications to surgery. Patients with stage 1-3 may be surgical candidates but not necessarily cured. MRI findings of focal thickening and enhancement of the interlobular fissures are useful signs in the detection of early malignant pleural disease.

How well did you know this?
1
Not at all
2
3
4
5
Perfectly
81
Q

A small pericardial metastatic deposit on the Magnetic Resonance Imaging (MRI) of a 55-year-old female demonstrates high-signal intensity on T1-weighted images. [Book 3 Question 19]

Which of the following is the most likely primary tumour?

A. Breast
B. Renal cell carcinoma
C. Colorectal
D. Melanoma
E. Lymphoma

A

Melanoma

Pericardial effusion, thickening or mass may indicate metastatic involvement of the pericardium. Most neoplasms have a low-signal intensity on T1-weighted images and a high signal intensity on T2-weighted images, except for metastatic melanoma. The most common tumours to metastasize to the pericardium are breast and lung, followed by lymphomas and melanomas.

82
Q

Which of the following is most likely to present as a haemorrhagic rather than a calcified pulmonary metastasis? [Book 3 Question 49]

A. Breast cancer
B. Chondrosarcoma
C. Testicular cancer
D. Choriocarcinoma
E. Mucinous adenocarcinoma of the colon

A

Choriocarcinoma

Choriocarcinoma, renal carcinoma, and melanoma are causes of haemorrhagic pulmonary metastases.

83
Q

A 34-year-old woman presents with a 4-month history of gradually increasing dyspnoea and cough. A CXR and subsequent CT scan show multiple cavitating lung lesions. On the CT scan, some of these lesions are noted to have surrounding ground-glass attenuation. No other abnormality is seen. Which of the following diagnoses are the findings most compatible with? [Book 1, Question 17]

A. Rheumatoid lung.
B. Lung abscesses.
C. Eosinophilic granuloma.
D. Churg–Strauss syndrome.
E. Melanoma metastases.

A

Melanoma metastases.

The GGO surrounding a nodule is known as the ‘halo’ sign and represents perilesional haemorrhage. Melanoma metastases can both cavitate and produce perilesional haemorrhage.

Differential diagnoses for Halo’s sign (Perilesional haemorrhage or cellular infiltration)
* Infection: Angio invasive fungi (classically Aspergillus, also Candida, Mucor), mycobacteria, rickettsia, viruses (e.g., Varicella zoster, Herpes simplex, Cytomegalovirus), and septic embolism
* Inflammatory: Granulomatosis with polyangiitis, eosinophilic pneumonia, cryptogenic organizing pneumonia, endometriosis
* Neoplastic: Kaposi sarcoma, pulmonary adenocarcinoma with lepidic features, vascular metastases (e.g., angiosarcoma, choriocarcinoma, osteosarcoma)
* Iatrogenic: Post-transbronchial biopsy, catheter-induced pulmonary pseudoaneurysm

Cavities with Halo
Wegener’s granulomatosis
Lymphoma,
Bronchoalveolar carcinoma
Squamous cell carcinoma
Haemorrhagic Mets

Cavities without Halo
Rheumatoid lung
Eosinophilic granuloma
Lung abscesses

Churg–Strauss syndrome is not associated with the halo sign and cavitation is rare.

84
Q

The staging chest CT of a 40-year-old man with a known primary malignancy demonstrates cavitating pulmonary metastases. The least likely type of primary lesion would be: [Book 2 Question 17]

a. Squamous cell carcinoma
b. Malignant melanoma
c. Renal cell cancer
d. Sarcomas
e. Colonic carcinoma

A

Renal cell cancer

RCC

85
Q

Which of the following has the greatest incidence of metastases to the lung? [Book 3 Question 7]

A. Choriocarcinoma
B. Hypernephroma
C. Ewing’s sarcoma
D. Rhabdomyosarcoma
E. Testicular seminoma

A

Choriocarcinoma

In order of decreasing frequency, pulmonary metastases arise from choriocarcinoma, hypernephroma/Wilms’ tumour, Ewing’s/rhabdomyosarcoma, and testicular tumours.

86
Q

Which is the most common primary tumour to metastasize to the pleura? [Book 3 Question 11]

A. Lung
B. Breast
C. Lymphoma
D. Ovary
E. Stomach

A

Lung

In order of decreasing frequency, metastases to the pleura arise from lung, breast, lymphoma, ovary and stomach cancers.

87
Q

A 16-year-old male presents with sudden shortness of breath. A chest radiograph shows multiple, bilateral nodules measuring up to 3 cm, some of which are calcified. There is a moderate left pneumothorax. The patient has been undergoing treatment for a malignant tumour. What is the most likely diagnosis? [Book 4 Question]

a. metastases secondary to Wilms’ tumour
b. metastases secondary to osteosarcoma
c. metastases secondary to testicular tumour
d. abscesses secondary to immunosuppression
e. varicella pneumonia secondary to immunosuppression

A

metastases secondary to osteosarcoma

Osteosarcoma pulmonary metastases are uncommon (seen in 2% of cases) and present as multiple masses which may calcify. There is a high incidence of associated pneumothorax. Wilms’ tumours may also produce multiple pulmonary masses and may be associated with pneumothorax but are not known to calcify. Testicular tumours may produce calcified lung metastases but are not associated with pneumothorax. Varicella pneumonia shows patchy consolidation in the acute phase, with multiple, small, calcified nodules in the chronic phase. Abscesses may present as multiple masses but rarely calcify and often cavitate.

88
Q

A 50-year-old male with known chronic asthma is seen in an outpatient clinic. He has no current symptoms. A ‘routine’ chest radiograph is performed. Which feature is most likely to be seen? [Book 4 Question 16]
a. hyper-expansion
b. peri-bronchial cuffing
c. bronchiectasis
d. parenchymal scars
e. normal chest radiograph

A

Normal chest radiograph
In chronic asthma, most patients (73%) have normal appearances on chest radiographs between acute episodes. Features such as bronchiectasis or parenchymal scarring may be seen, especially with episodes of repeated infection. Hyper-expansion and peri-bronchial cuffing are features seen during acute exacerbations of asthma.

89
Q

A 35-year-old male smoker presents with a history of progressive dyspnoea and rapidly deteriorating lung function. CXR shows hyperinflated lungs and decreased pulmonary vascular markings. High-resolution CT of chest shows well-defined foci of reduced lung attenuation without definable wall, decreased pulmonary vascular markings and bullae with basilar predominance. What is the likely diagnosis? [Book 1 Question 38]
A. Centrilobular emphysema.
B. Para-septal emphysema.
C. Alpha-1 antitrypsin deficiency.
D. Congenital lobar emphysema.
E. Chronic obstructive pulmonary disease (COPD).

A

Alpha-1 antitrypsin deficiency
Alpha-1 antitrypsin is a glycoprotein synthesized in the hepatocytes, which acts as a proteolytic inhibitor. In the absence of alpha-1 antitrypsin, the enzyme elastase released by neutrophils and alveolar macrophages acts unopposed and digests the basement membrane. There is rapid progressive deterioration of lung function due to severe pan-acinar emphysema, which shows basilar predominance (due to gravitational distribution of pulmonary blood flow). The alveolar destruction is accelerated in smokers. Cirrhosis of the liver is a complication.

90
Q

A young man presents with progressive productive cough and halitosis. He had severe pneumonia as a child. Plain chest radiograph demonstrates bronchial dilatation and bronchial wall thickening with some volume loss. Which of the following HRCT findings is the most sensitive finding for bronchiectasis? [Book 2 Question 31]
a. Air trapping
b. Mucous-filled dilated bronchi
c. Bronchial wall thickening
d. Bronchi seen in the subpleural region
e. Lack of bronchial tapering

A

Lack of bronchial tapering
Whilst all the above can be seen in patients with bronchiectasis, a lack of progressive tapering of the bronchi is the most sensitive (80%).

91
Q

A 56-year-old man presents with shortness of breath. He subsequently has an HRCT of chest performed. This shows a mosaic attenuation pattern throughout the lung parenchyma, but you are having some difficulty determining if the more lucent areas are normal or abnormal. Which of the following findings is most likely to be helpful in confirming that the lucent areas are the abnormal areas, and you are not dealing with multifocal GGO? [Book 1 Question 52]
A. Increased calibre of vessels in denser areas.
B. Decreased calibre of vessels in denser areas.
C. Increased calibre of vessels in lucent areas.
D. Decreased calibre of vessels in lucent areas.
E. Calibre of vessels is unhelpful and expiratory scans must be used.

A

Decreased calibre of vessels in lucent areas
In multiple focal mosaic lung attenuation
* If the blood vessels in the lucent areas are smaller, then the lucent areas are probably abnormal.
o The paucity of vessels in these regions may be secondary to focal air-trapping or poor ventilation and subsequent reflex vasoconstriction. If areas of lucency are exaggerated on expiratory scans (air-trapping), then this is the hallmark of small airways disease.
o Alternatively, if the areas of lucency do not become more prominent on the expiratory scans, small airways disease is not the likely cause. In this situation, the inhomogeneous lung attenuation is probably secondary to changes in vessel calibre, and secondary to pulmonary hypertension, including chronic PE, emphysema, or inflammatory vasculopathies.
* If the blood vessels in the regions of relative lucency are equal in size to vessels in surrounding areas, the regions of relative opacity are most likely abnormal, e.g., areas of GGO.

92
Q

A 56-year-old female patient presents with shortness of breath. A chest radiograph is unremarkable. A high-resolution CT scan is performed which shows mosaic perfusion with no air trapping on expiratory scan. What is the most likely diagnosis? [Book 4 Question 4]
a. bronchiolitis obliterans
b. cystic fibrosis
c. hypersensitivity pneumonitis
d. chronic thromboembolic disease
e. asthma

A

Chronic thromboembolic disease
Mosaic perfusion is caused by abnormalities of ventilation, or vascular obstruction. Expiratory scans help to distinguish causes by establishing whether there is air trapping. With no air trapping present, pulmonary emboli of any cause are most likely. Air trapping would suggest airway disease such as bronchiolitis obliterans, or other causes of small airway obstruction such as bronchiectasis or cystic fibrosis.

93
Q

On high-resolution CT of the chest, how can mosaic perfusion be distinguished from ground-glass opacification? [Book 4 Question 97]
a. the opaque areas have abnormally small vessels
b. the lucent areas have abnormally small vessels
c. mosaic perfusion is restricted to lower zones
d. mosaic perfusion is restricted to subpleural regions
e. cannot be reliably distinguished

A

The lucent areas have abnormally small vessels
Mosaic perfusion refers to areas of altered attenuation on CT and reflects vascular obstruction or abnormal ventilation. Differentiation from ground-glass opacification can be difficult but can usually be done by assessment of the centrilobular vessels. In comparing ‘lucent’ and ‘opaque’ areas, the area in which the vessels appear larger is generally the normal area, though this may be a difficult differentiation to make in practice, as the vessels may be of the same size throughout the lung. In mosaic perfusion, the vessels appear abnormal in the ‘lucent’ area, whereas, in ground-glass opacification, the vessels appear abnormal in the ‘opaque’ area. Expiratory scans may then help to distinguish airway causes from vascular causes, by the presence of air trapping, which is seen with airway disease.

94
Q

An 18-year-old male presents with a chest radiograph performed for immigration purposes. He is noted to have dextrocardia. Further investigation reveals nasal polyposis and bronchiectasis. Which further investigation should be considered? [Book 4 Question 79]

a. renal function
b. CT of brain
c. fertility assessment
d. renal angiogram
e. thyroid function test

A

Fertility assessment

The presence of dextrocardia, nasal polyposis and bronchiectasis raises the possibility of Kartagener’s syndrome. This is immotile/dysmotile cilia syndrome and may also present with deafness and infertility. There is also an association with transposition of the great vessels, pyloric stenosis, post-cricoid web and epispadias.

95
Q

A 45-year-old male smoker has a 6-month history of gradually increasing shortness of breath and cough. CXR shows a mild increase in interstitial markings in the mid and upper zones. A HRCT is requested for clarification, and this demonstrates ill-defined centrilobular ground-glass nodules, more pronounced in the mid and upper zones. There is no traction bronchiectasis or honeycombing. What is the most likely diagnosis? [Book1 Question 3]

A. Desquamative interstitial pneumonia (DIP).
B. Usual interstitial pneumonia (UIP).
C. Respiratory bronchiolitis interstitial lung disease (RBILD).
D. Non-specific interstitial pneumonia (NSIP).
E. Cryptogenic organizing pneumonia (COP).

A

Respiratory bronchiolitis interstitial lung disease (RB-ILD).

RBILD is a disease of smokers and the centrilobular nodules reflect chronic inflammation in the respiratory bronchioles. GGO may occur and is typically multifocal and upper lobe predominant. Reticulation is uncommon and fibrosis is not a typical feature. The typical features of UIP are reticulation, honeycombing, and traction bronchiectasis, with a basal and subpleural predominance. NSIP has overlapping features with UIP, but GGO is more common and honeycombing less common. DIP is a rare disease and, like RBILD, associated with smoking. The cardinal feature is GGO, with a basal and peripheral predominance. Sometimes small cystic areas can be seen within the areas of GGO. Progression to fibrosis/honeycombing is rare. COP was formerly known as bronchiolitis obliterans organizing pneumonia (BOOP). The main finding on HRCT is consolidation, which is usually multifocal and bilateral. It typically has a lower zone and peripheral predominance but can affect any lobe. Dilated airways are often seen on HRCT with air bronchograms mimicking acute pneumonia. There may also be GGO and reticulation, but lung volumes are usually maintained.

96
Q

Which of the following features detected on HRCT would most favour NSIP rather than USIP? [Book 3 Question 43]

A. Predominant ground-glass opacification
B. Honeycombing
C. Predominant lower zone disease
D. Traction bronchiectasis
E. Subpleural predominance

A

Predominant ground-glass opacification

Although Ground glass opacification (GGO) can be commonly seen in USIP, it is found in 75-100% of cases of NSIP, where it is usually bilateral, symmetrical and subpleural in two-thirds of patients and has a lower zone predominance on over 50%. Honeycombing is not a major feature, although it can occur in up to 50%. Typical HRCT features of USIP include reticulation, honeycombing and traction bronchiectasis with basal and subpleural predominance.

97
Q

A 56-year-old woman with a history of Sjogren’s syndrome complains of gradually increasing shortness of breath. A CXR has identified a mild generalized interstitial pattern, with maintained lung volumes. A subsequent HRCT of chest demonstrates a few scattered well-defined, regular lung cysts. Within the lung parenchyma there is also noted patchy ground-glass change and mild centrilobular nodularity. Mild mediastinal and hilar lymphadenopathy is present. What is the most likely diagnosis? [Book 1 Question 32]

A. Langerhans cell histiocytosis.
B. Desquamative interstitial pneumonia.
C. Lymphangioleiomyomatosis.
D. Lymphocytic interstitial pneumonia.
E. Birt–Hogg–Dube syndrome.

A

Lymphocytic interstitial pneumonia.

LIP is a benign lymphoproliferative disorder that is also associated with AIDS, autoimmune thyroid disease, and Castleman’s syndrome. Lymphomas may arise in some cases. On HRCT: * Randomly distributed thin-walled cysts. * GGO, centrilobular/subpleural nodules, septal thickening. * Eventually larger nodules (>2 cm), consolidation, and architectural distortion may develop. * Mediastinal and hilar lymphadenopathy (2/3) o The presence of mediastinal and hilar lymphadenopathy (two-thirds) and septal thickening help distinguish from Langerhans cell histiocytosis. o The presence of centrilobular nodules assist in the differentiation from LAM. Birt–Hogg–Dube syndrome: * AD * Facial fibrofolliculomas * Malignant renal tumours * Thin-walled pulmonary cysts * Spontaneous pneumothorax.

98
Q

A 45-year-old male presents with a 3-month history of a non-productive cough and dyspnoea, which was preceded by a flu-like illness. Pulmonary function tests reveal a restrictive pattern and a CXR shows multifocal bilateral consolidation. HRCT of chest reveals bilateral peripheral subpleural well-defined areas of consolidation, some of which are surrounded by ground-glass opacity and some of which show an air bronchogram. There is also a focal area of GGO, which is surrounded by a smooth-walled ring of consolidation. Which of the following is the most likely diagnosis? [Book 1 Question 45]

A. Sarcoidosis.
B. TB.
C. Cryptococcosis.
D. Obliterative bronchiolitis.
E. Cryptogenic organizing pneumonia.

A

Cryptogenic organizing pneumonia.

An area of GGO surrounded by a ring of consolidation describes the ‘ring halo’ sign. This is not specific to COP, but has been described in tuberculosis, active sarcoidosis, cryptococcosis, and blastomycosis. However, it has been found that granulomatous infectious diseases and sarcoidosis, which cause the ‘ring halo’ sign, result in a nodular ring, whereas COP results in a smooth-walled ring. COP is rapidly responsive to steroids, but the latter can have deleterious effects in infectious diseases. Thus, it has been suggested that a nodular ring can be used as a discriminator. COP (also known as idiopathic BOOP) is a patchy organizing pneumonia caused by bronchiolar obstruction by plugs of loose organizing connective tissue that may wax and wane. The main finding of COP is consolidation, seen in 90% and usually multifocal and bilateral. In 50% the consolidation is subpleural or peri-broncho-vascular. It has a lower zone predominance but can affect any lobe. Dilated airways are often seen on CT and air bronchograms give the appearance of acute pneumonia. GGO is a prominent pattern in 60%. Linear opacities occur in isolation or in association with multifocal areas of consolidation. Nodules are seen in 30–50% and lung volumes are preserved in 75%. COP is a clinical-pathological entity and is diagnosed when the correct clinical picture and radiological findings are present. If these features are uncertain, lung biopsy is required. Organizing pneumonia can be idiopathic (COP) or secondary to viral infection, toxic fume inhalation, RA, systemic lupus erythematosus (SLE), organ transplantation, drug reaction, or chronic aspiration. The hallmark of obliterative bronchiolitis is air-trapping and hyperinflation. It has many causes, including RA and other connective tissue diseases.

99
Q

A 50-year-old woman presents with gradually increasing shortness of breath. A CXR and HRCT of chest show subpleural reticulation, more marked in the lower zones. Which of the following further findings on HRCT is most likely to support the diagnosis of NSIP? [Book 1 Question 7]
A. Centrilobular nodules.
B. Air-trapping.
C. GGO.
D. Cystic changes.
E. Pleural effusions.

A

GGO.
This is a salient feature of NSIP and is seen in almost all cases. Diffuse nodules are very infrequent in NSIP. If centrilobular nodules are present, one should think of other forms of diffuse lung disease such as respiratory bronchiolitis interstitial lung disease (RBILD) or hypersensitivity pneumonitis. If multiple cysts are present, again other diffuse lung disease should be considered, such as lymphocytic interstitial pneumonia (LIP), DIP, lymphangioleiomyomatosis, and pulmonary Langerhans cell histiocytosis. Areas of air-trapping would be a more typical finding in hypersensitivity pneumonitis, rather than NSIP. Pleural effusions are not a typical finding in NSIP.

100
Q

Which of the following favours Hughes-Stovin syndrome rather than Behçet’s syndrome? [Book 3 Question 41]
A. Diffuse airspace nodularity
B. Pulmonary consolidation
C. Pulmonary artery aneurysm
D. A lack of oral/genital ulcers
E. Mosaic perfusion on CT

A

A lack of oral/genital ulcers
Hughes-Stovin syndrome, also known as incomplete Behçet’s disease, overlaps in radiologic and histopathologic findings, but there is an absence of oral and genital ulcerations.

101
Q

A 45-year-old male patient is referred by his GP for a CXR due to a history of dyspnoea and cough. The CXR shows a convex appearance to the hila with a right paratracheal stripe that measures 1.5 cm. You are arranging follow-up and the respiratory team ask you for your top differential. What do you say? [Book 1 Question 59]
A. TB.
B. Lymphoma.
C. Sarcoid.
D. Castleman’s disease.
E. Silicosis.

A

Sarcoid.
There are numerous causes of bilateral hilar lymphadenopathy (BHL) and in the absence of further clinical information, sarcoid would always feature high in the list of differentials. However, the finding of BHL in association with paratracheal adenopathy is classical for this disease.

102
Q

The findings of a HRCT scan include irregular broncho-vascular, interlobular septal and pleural thickening. There is beading along the broncho-vascular bundles, traction bronchiectasis and areas of ground-glass opacification. Which is the most likely diagnosis? [Book 3 Question 15]
A. Tuberculosis
B. Sarcoid
C. Lymphoma
D. Usual Interstitial Pneumonitis (UIP)
E. Non-specific Interstitial Pneumonitis (NSIP)

A

Sarcoid
These are characteristic features of sarcoid on HRCT. Peri-lymphatic nodules (beading along the fissures) and architectural distortion are commonly found.

103
Q

A 33-year-old female patient presents with a longstanding history of fever, dry cough, and weight loss. The chest radiograph reveals mediastinal lymphadenopathy. Blood investigations reveal hypercalcemia and elevated angiotensin-converting enzyme (ACE). Which of the following appearances of lymphadenopathy on CT would be the least likely feature in favour of the clinical diagnosis? [Book 2 Question 43]
a. Bilateral hilar lymphadenopathy
b. Egg-shell calcification
c. Predominant involvement of the right paratracheal lymph nodes
d. Lymphadenopathy without any parenchymal involvement
e. Posterior mediastinal lymph nodes

A

Posterior mediastinal lymph nodes
All the above are features of mediastinal lymphadenopathy in sarcoidosis. Whilst sarcoidosis can involve different mediastinal and hilar groups, posterior mediastinal lymphadenopathy is a feature of NHL.

104
Q

A 45-year-old male smoker has a history of fatigue and mild shortness of breath. He also keeps pigeons. A CXR shows mildly increased interstitial markings in the upper zones. An HRCT of chest demonstrates multiple small pulmonary nodules and reticulation, more marked in the upper lungs. What location of the nodularity is more likely to suggest a diagnosis of subacute extrinsic alveolitis or respiratory bronchiolitis interstitial lung disease as opposed to sarcoidosis? [Book 1, Question 22]
A. Broncho-vascular bundle.
B. Centrilobular region.
C. Fissure.
D. Subpleural region.
E. Interlobular septa.

A

Centrilobular region.
In sarcoid, the granulomatous nodules are typically distributed along the lymphatics and are therefore seen along the bronchovascular bundles, interlobular septa, major fissures, and subpleural regions. The centrilobular region of the secondary pulmonary lobule contains the bronchiole and therefore conditions that cause peribronchiolar inflammation, such as subacute EAA and RB-ILD more often cause centrilobular nodules. These are often ground glass in attenuation and ill-defined, unlike the more solid appearing granulomatous nodules of sarcoid. Rarely sarcoid can cause centrilobular nodules because of the presence of peribronchiolar granulomas, but the other mentioned locations are much more typical.

105
Q

A 50-year-old male presents with a history of occasional haemoptysis and exertional shortness of breath which has been getting progressively worse. Plain chest radiograph demonstrates bi-basal reticular shadowing with volume loss. HRCT demonstrates bi-basal fibrosis and traction bronchiectasis. Incidental note is made of a patulous oesophagus. Which of the following is the most likely cause? [Book 2 Question 1]

a. Tuberculosis
b. SLE
c. Rheumatoid arthritis
d. Wegener’s granulomatosis
e. Scleroderma

A

Scleroderma

Whilst haemoptysis may be a presentation in tuberculosis and Wegener’s and bi-basal fibrosis maybe seen in all the above except tuberculosis (where apical fibrosis is the more likely feature), scleroderma is the only condition resulting in a patulous lower oesophageal sphincter, oesophageal shortening, and stricture formation.

106
Q

Which of the following features is most strongly associated with active alveolitis on HRCT in a 40-year-old woman with a diagnosis of systemic sclerosis? [Book 3 Question 32]

A. Ground glass opacification without traction bronchiectasis in the middle lobe
B. Basal ground glass opacification
C. Lower lobar consolidation
D. Interlobular septal thickening
E. Pleural effusion

A

The presence of ground glass opacification (without evidence of traction bronchiectasis) in the middle lobe and lingula is strongly associated with active alveolitis. Basal ground glass shadowing is a non-specific finding and a poor predictor of alveolitis.

107
Q

A 26-year-old female patient with an optic nerve tumour and café-au-lait spots presents with exertional breathlessness. Imaging of the chest is most likely to reveal which of the following? [Book 2 Question 34]

a. Multiple small lower lobe cysts
b. Emphysema
c. Lower zone fibrosis
d. Thick-walled cavities in the upper zone
e. Asymmetrical upper zone fibrosis

A

Lower zone fibrosis

The case describes neurofibromatosis I, which is associated with lower zone fibrosis and thin-walled bullae, mainly in the upper zones. Apart from the pulmonary changes, skeletal abnormalities involving the ribs and spine and mediastinal masses may also be seen.

108
Q

A 51-year-old man with long standing history of an erosive arthropathy of the acromioclavicular joints and bilateral arthropathy in his hands subsequently develops progressive shortness of breath. The most likely abnormality on his chest radiograph would be: [Book 2 Question 11]

a. Cavitating nodules
b. Peripheral basal reticulonodular shadowing
c. Cardiomegaly
d. Bronchiectasis
e. Pleural effusion

A

Pleural effusion

Pleural effusion is the commonest thoracic manifestation of rheumatoid arthritis, much more common in men (M
= 9:1). It is unilateral in most cases. The fluid is an exudate with low sugar content and is often seen in the absence of other pulmonary changes.

109
Q

A 25-year-old woman with a longstanding history of non-erosive arthritis of the hands and a malar rash presents with progressive breathlessness and respiratory dysfunction. Blood serology demonstrates anti-DNA antibodies. Which of the following is the most common feature on the chest radiograph? [Book 2 Question 18]

a. Pleural effusion
b. Consolidation
c. Cavitating nodules
d. Pulmonary oedema
e. Pulmonary fibrosis

A

Pleural effusion

Whilst all the above are seen in SLE, pleural effusions are the commonest radiographic abnormality.

110
Q

In patients with rheumatoid arthritis, what is the commonest pulmonary finding seen on the chest radiograph? [Book 4 Question 56]

a. pleural effusion
b. fibrosis
c. pulmonary nodules
d. bronchiectasis
e. heart failure

A

Pleural Effusion

Between 2% and 54% of patients with rheumatoid arthritis have pulmonary abnormalities. Pleural abnormalities are most frequent, being either an effusion (unilateral in 92% of cases) or pleural thickening (usually bilateral). Fibrosis occurs in 30% of patients with pulmonary involvement. Nodules are unusual and seen in advanced disease. They are usually peripheral and may cavitate. Bronchial abnormalities are seen in 30% of patients with rheumatoid lung and include bronchiectasis and bronchiolitis obliterans. Other findings include pulmonary arterial hypertension and heart failure secondary to carditis/pericarditis.

111
Q

A patient with a known collagen vascular disease has pulmonary fibrosis. HRCT reveals bilateral lower lobe bronchiectasis. Which collagen vascular disease is most likely? [Book 2 Question 49]

a. Sjogren syndrome
b. Progressive systemic sclerosis
c. SLE
d. Rheumatoid arthritis
e. Dermatomyositis

A

Sjogren syndrome

Whilst pulmonary fibrosis is a feature of all the above conditions, bronchiectasis is most likely seen in Sjogren syndrome.

112
Q

A 28-year-old man is being investigated for haemoptysis. He has a history of sinusitis. Full blood picture is normal. He is referred for a CT of chest during which intravenous (IV) contrast was withheld by the radiographer due to a reduction in estimated glomerular filtration rate (eGFR). It reveals bilateral nodules in a peribronchovascular distribution, some of which show cavitation. There are peripheral wedge-shaped areas of consolidation. There are also areas of bronchial stenosis and thickening. No mediastinal or hilar adenopathy is present. What is the most likely diagnosis? [Book 1, Question 15]

A. Goodpasture’s syndrome.
B. Sarcoidosis.
C. Churg–Strauss syndrome.
D. Wegener’s granulomatosis.
E. Pulmonary tuberculosis.

A

Granulomatosis with Polyangiitis

This condition is associated with the findings described, including bilateral nodules and cavitation, particularly in the context of a history of sinusitis.

113
Q

A 40-year-old male presents with shortness of breath. He also has lower back pain and stiffness of the spine. A chest radiograph shows bilateral upper-zone fibrosis with elevation of the hila. Spinal ligamentous ossification is also noted. High-resolution CT shows peripheral interstitial changes with traction bronchiectasis and para-septal emphysematous changes in the upper zones. What is the most likely diagnosis?
a. ankylosing spondylitis
b. Reiter’s syndrome
c. tuberculosis
d. sarcoidosis
e. chronic extrinsic allergic alveolitis

A

Ankylosing spondylitis

Pulmonary changes occur in ankylosing spondylitis in 1% of patients, with reticulonodular abnormality in the upper zones progressing to confluent opacification and fibrosis. There may be apical bullous formation and cavitation mimicking tuberculosis. No granulomatous disease is seen. Ossification of spinal ligaments is typical in ankylosing spondylitis. Upper-zone fibrosis is seen with tuberculosis, sarcoidosis, and chronic extrinsic allergic alveolitis, but spinal ossification is not a feature of these conditions. The reciprocal is true of Reiter’s syndrome.

114
Q

In systemic sclerosis, what is the most common pulmonary finding on CT?
a. consolidation secondary to aspiration pneumonia
b. pulmonary hypertension
c. pleural effusions
d. interstitial lung disease
e. pulmonary emboli

A

Interstitial lung disease

All of these features can occur, but 65% of patients have fibrosis on CT, although only 30% of patients with systemic sclerosis have clinically significant lung disease. The commonest interstitial lung disease is nonspecific interstitial pneumonitis, seen in two-thirds of cases. This is less coarse and extensive than usual interstitial pneumonitis, the other type seen, and is almost always associated with ground-glass opacification. Other respiratory features seen in systemic sclerosis are organizing pneumonia, pulmonary haemorrhage, pleural thickening, and malignancy.

115
Q

A 68-year-old patient presents with cough and dyspnoea. A chest radiograph is performed. Which feature would suggest sarcoidosis as a more likely diagnosis than tuberculosis?
a. pleural effusion
b. cavitating upper lobe lesion
c. calcified lung lesion
d. symmetrical hilar lymphadenopathy
e. consolidation

A

Symmetrical hilar lymphadenopathy

Differentiation of tuberculosis and sarcoidosis can be difficult clinically and radiologically. The presence of symmetrical hilar lymphadenopathy is seen more often with sarcoidosis than with tuberculosis. Pleural effusions, and cavitating and calcified lesions are much more common with tuberculosis. Consolidation can be seen in both conditions, though it is again more commonly seen in tuberculosis.

116
Q

A 60-year-old female presents with increasing shortness of breath. She is known to have rheumatoid arthritis. A chest radiograph shows reticulonodular changes, and high-resolution CT is performed. Which feature would suggest underlying sarcoidosis as a more likely diagnosis than rheumatoid lung?
a. lower-zone predominance of reticulation
b. mid-zone predominance of reticulation
c. pleural effusion
d. multiple nodules larger than 20 mm
e. cardiomegaly

A

Mid-zone predominance of reticulation

Sarcoidosis is seen more commonly in females and affects the thorax in 90% of cases. It usually produces adenopathy with or without parenchymal disease. There is a mid-zone predominance with irregular septal thickening, peri-lymphatic nodules, ground-glass opacification, traction bronchiectasis, and honeycombing. Rheumatoid lung is more common in males and mostly presents with pleural abnormalities, usually an effusion. It may present with an interstitial fibrotic picture with a lower-zone predominance. Multiple large nodules of up to 7 cm in diameter are seen with rheumatoid lung. Cardiomegaly may be seen with both sarcoidosis and rheumatoid, because of congestive cardiac failure.

117
Q

A 36-year-old asthmatic attends an outpatient respiratory clinic complaining of recent increasing dyspnoea. Bloods show an elevated white cell count and eosinophilia. Chest radiograph reveals multiple areas of ill-defined peripherally based consolidation. Subsequent chest radiographs over the coming week show the consolidation to resolve in places but commence in other previously unaffected areas. The most likely cause is:
a. Alveolar sarcoidosis
b. Bronchioalveolar carcinoma
c. Acute eosinophilic pneumonia
d. Chronic eosinophilic pneumonia
e. Loffler syndrome

A

Loffler syndrome

This is described as simple eosinophilic pneumonia but is of unknown aetiology. Pathologically, there is interstitial and alveolar oedema. Patients are usually asthmatic/atopic but have mild or no symptoms. Classical chest radiograph appearance is of fleeting infiltrates with transient and shifting peripheral consolidation.

118
Q

The following finding is more in keeping with chronic eosinophilic pneumonia rather than Churg-Strauss:
a. Centrilobular nodules within areas of ground-glass change
b. Peripheral consolidation with tendency towards lobular distribution
c. Transient opacities, shifting over days
d. Homogenous peripheral airspace consolidation
e. Interlobular septal thickening

A

Homogenous peripheral airspace consolidation

Chronic eosinophilic pneumonia is characterised by the presence of homogenous peripheral airspace consolidation on CT, whereas the peripheral consolidation in Churg-Strauss has a tendency towards lobular distribution and is often associated with centrilobular nodules within the ground glass opacity.

119
Q

A 43-year-old female with Churg–Strauss syndrome has a high-resolution CT. What is the most likely finding?
a. small centrilobular nodules
b. bullous disease
c. bilateral, symmetrical, hilar lymphadenopathy
d. tumour-like mass
e. pleural effusion

A

Small centrilobular nodules

Churg–Strauss syndrome is rare, though pulmonary abnormalities are a common feature of the condition. Typically, airspace and airway patterns are seen with the following features – centrilobular nodules, ground-glass opacities, bronchial wall thickening, bronchiectasis, consolidation, and septal thickening. Lymphadenopathy may be seen in the mediastinum and at the hila but is not a common feature. Large nodules may occur, but a tumour-like mass is not a feature. Pleural effusions and bullae are not seen.

120
Q

Which of the following is most likely to show peripheral reverse bats wings infiltrates on a chest radiograph? [Book 3 Question 46]
A. Eosinophilic pneumonia
B. Pulmonary oedema
C. Proteinosis
D. Lymphoma
E. Goodpasture’s syndrome

A

Eosinophilic pneumonia, resolving pulmonary oedema, desquamative interstitial pneumonia and sarcoidosis are causes of reverse bats wings peripheral infiltrates.

Causes of perihilar bats wings infiltrates include pulmonary oedema, proteinosis, sarcoidosis, lymphoma, Goodpasture’s syndrome and alveolar cell carcinoma.

121
Q

A 43-year-old patient presents with cough, shortness of breath and fever which has lasted a month. An HRCT reveals bilateral areas of consolidation, predominantly in a peripheral distribution. There are also areas of GGO, predominantly in the middle and upper zones, with band-like subpleural attenuation. The plain film findings have remained unchanged for days. What is the most likely diagnosis? [Book 1, Question 30]
A. Chronic eosinophilic pneumonia.
B. Allergic bronchopulmonary aspergillosis (ABPA).
C. Acute eosinophilic pneumonia.
D. Loffler’s syndrome.
E. Eosinophilic granuloma.

A

Chronic eosinophilic pneumonia.

  • The radiological ‘photographic negative’ of pulmonary oedema.
  • Middle age, females > males.
  • Common history of atopy
  • Histology – filling of alveolar spaces with inflammatory eosinophilic infiltrates in alveolar
    spaces, and in interstitium and peripheral blood eosinophilia.
  • A dramatic response to steroid therapy within days.

ABPA and acute eosinophilic pneumonia

do produce blood eosinophilia
ABPA – bronchiectasis and mucus plugging with the possibility of mosaic perfusion in
addition to peripheral consolidation,
Acute eosinophilic pneumonia – diffuse GGO, defined nodules, smooth interlobular septal
thickening, and often the presence of pleural effusion.

Simple pulmonary eosinophilia = Löffler’s syndrome

Peripheral transient parenchymal consolidation accompanied by eosinophilia.
Minimal or no pulmonary symptoms
CXR appearances change within one to several days and
spontaneous resolution occurs within one month.

Eosinophilic granuloma is nodular/cystic, a pulmonary form of Langerhan’s cell histiocytosis,
and should not be confused with the eosinophilic pneumonias.

122
Q

A 42-year-old male presents with stridor and persistent cough. He previously has had several nosebleeds. Bloods show mild renal impairment. A chest radiograph shows multiple cavitary lesions with irregular lining, predominantly in the lower lobes. What is the most likely diagnosis? [Book 4 Question 14]
a. metastatic disease from nasopharyngeal carcinoma
b. Wegener’s granulomatosis
c. sarcoidosis
d. pyogenic abscesses
e. systemic lupus erythematosus

A

Wegener’s granulomatosis

The combination of nosebleeds and cavitatory nodules is very suggestive of Wegener’s granulomatosis, especially with renal impairment. In the lungs there is often patchy alveolar infiltration, pleural effusions and cavitatory nodules. Systemic lupus erythematosus produces similar appearances by the same mechanism of vasculitis. Sarcoidosis produces many lung changes but cavitatory nodules are rare. Metastatic nasopharyngeal carcinomas are usually squamous cell tumours and can produce cavitatory metastases, but primary tumours usually present with nasal obstruction, although they may cause epistaxis. Renal impairment is not a feature. Multiple abscesses are not usually associated with epistaxis and present acutely, typically in intravenous drug users.

123
Q

A 45-year-old female presents with malaise and cough. She has a history of multiple allergies. Her blood results show an eosinophilia, and the chest radiograph reveals two areas of peripheral consolidation. A further chest radiograph 2 days later shows these to be resolving. Which of the following is the most likely diagnosis? [Book 4 Question 27]
a. histiocytosis
b. pseudomonas pneumonia
c. Klebsiella pneumonia
d. Loeffler’s syndrome
e. lipoid pneumonia

A

Loeffler’s syndrome

Loeffler’s syndrome is of unknown aetiology and is characterized by areas of non-segmental consolidation, which are peripherally situated and transient. Histiocytosis produces ill-defined nodules which cavitate, first producing thick-walled and then thin-walled cysts. Klebsiella pneumonia usually affects the upper lobes, producing dense, lobar consolidation, sometimes with bulging fissure and empyema. Pseudomonas pneumonia is patchy but extensive and usually in the lower lobes. Lipoid pneumonia is segmental and homogeneously dense, and changes only slowly.

124
Q

A 35-year-old with asthma presents with malaise, flu-like illness and cough. Previous similar episodes have occurred. A chest radiograph shows patchy airspace opacification in the mid and upper zones. Which feature on high-resolution CT would make allergic bronchopulmonary aspergillosis a more likely diagnosis than extrinsic allergic alveolitis? [Book 4 Question 43]
a. widespread centrilobular micronodules, 3mm
b. tubular finger-like opacities
c. bronchiectasis
d. upper-zone fibrosis
e. pleural effusion

A

Tubular finger-like opacities

Allergic bronchopulmonary aspergillosis (ABPA) is hypersensitivity to aspergillus in people with asthma. Typical features are of a migratory pneumonitis, predominantly in the upper lobes. It may cause bronchiectasis and upper-zone fibrosis, which are features also seen in extrinsic allergic alveolitis (EAA). Tubular opacities, indicating mucus plugging, are seen in ABPA, but not in EAA. Centrilobular nodules are seen in EAA, along with mosaic perfusion and patchy ground-glass change. Pleural effusions are rarely seen in EAA and not in ABPA.

125
Q

A 63-year-old male has a complex past medical history including testicular carcinoma, cardiac disease, rheumatoid arthritis (RA) and diabetes mellitus. He presents with shortness of breath and is referred for a CT chest. This reveals multiple areas of ground-glass attenuation, crazy-paving, and consolidation in both lungs. You also notice that the spleen and liver are of increased attenuation. What is the most likely explanation for these findings? [Book 1 Question 40]
A. RA-related lung disease.
B. Cardiac failure.
C. Amiodarone.
D. Bleomycin.
E. Methotrexate.

A

C. Amiodarone.

The pulmonary findings could equally be caused by NSIP and cardiac failure, among other causes. However, the high attenuation of the liver and spleen is due to deposition of amiodarone, which contains iodine.

Amiodarone, methotrexate, and bleomycin all may cause pulmonary toxicity. Pulmonary toxicity occurs in 5–10% of patients on amiodarone, usually within months of starting therapy. The prognosis is good, with most patients improving after discontinuation of therapy. NSIP is the most common manifestation of amiodarone-induced lung disease. Pleural inflammation is an accompanying feature and can manifest as pleural effusion. COP is less common and typically occurs in association with NSIP. A distinctive feature of amiodarone toxicity is the occurrence of focal, homogenous pulmonary opacities. They are typically peripheral in location and of high attenuation at CT, due to the incorporation of amiodarone into the type II pneumocytes. The combination of high attenuation within the lung, liver, and spleen is characteristic of amiodarone toxicity.

RA can produce many pulmonary sequelae: pulmonary nodules, pleural effusion, fibrosis, obliterative bronchiolitis, and COP. Methotrexate can be used to treat RA and psoriasis, and as chemotherapy for various cancers. NSIP is most common, and COP is seen less frequently.

Bleomycin is a chemotherapy agent used in the treatment of testicular carcinoma, among others. Diffuse alveolar damage (DAD) is its most common manifestation, with NSIP and COP being less common. The prognosis is poor, with most patients dying of respiratory failure within 3 months of the onset of symptoms.

126
Q

A 64-year-old woman presents with a 3-week history of dry cough. A CXR is performed and shows multifocal bilateral peripheral areas of consolidation. An HRCT of chest is recommended, and this demonstrates bilateral peripheral areas of consolidation and GGO. There is no fibrosis. There is peripheral eosinophilia detected on routine blood tests. A careful drug history is obtained. Which of the following medications is the patient most likely to be on? [Book 1 Question 65]
A. Nitrofurantoin.
B. Amiodarone.
C. Methotrexate.
D. Bleomycin.
E. Cyclophosphamide.

A

A. Nitrofurantoin.

Drugs causing pulmonary eosinophilia
1. Nitrofurantoin
2. Penicillamine
3. Sulphasalazine
4. NSAID, and para-aminosalicylic acid.

Drugs associated with diffused alveolar damage
Bleomycin and cyclophosphamide, busulphan, carmustine, gold salts, and mitomycin.

In early DAD, HRCT typically shows scattered or diffuse areas of GGO. Fibrosis typically develops within 1 week and if progressive can cause marked architectural distortion and honeycombing.

Amiodarone and methotrexate are associated with an NSIP pattern on HRCT. Carmustine and chlorambucil can also cause this appearance. With early disease, HRCT scans may show only scattered or diffuse areas of GGO. Later, findings of fibrosis (traction bronchiectasis, honeycombing) predominate in a basal distribution.

Other recognized patterns of drug-induced lung injury include a COP pattern and diffuse pulmonary haemorrhage.

127
Q

A 61-year-old man has a history of chronic myeloid leukaemia. He presents with mild dyspnoea and dry cough. A CXR shows symmetrical, perihilar reticulo-nodular opacities with relative sparing of the apices and costophrenic angles, without cardiomegaly or pleural effusion. An HRCT reveals smoothly thickened septal lines with intervening GGO and sharply marginated areas of geographic sparing. Bronchoalveolar lavage (BAL) is negative for organisms. What is the most likely diagnosis? [Book 1, Question 25]
A. Pulmonary haemorrhage.
B. Left ventricular failure.
C. Pulmonary alveolar proteinosis.
D. Pneumocystis pneumonia.
E. Radiation fibrosis.

A

C. Pulmonary alveolar proteinosis.

This is rare, but most commonly develops in a primary idiopathic form (90%), chiefly in middle
aged smokers, with a male predominance. It can also occur secondary to industrial dust
exposure, immunodeficiency disorders (HIV and iatrogenic causes), and, as in the question,
secondary to underlying haematological malignancies. There is also a congenital form.

The radiological findings are often out of proportion with the disease severity. The HRCT
findings described are those of ‘crazy-paving’. There is a wide differential diagnosis for this,
including pulmonary oedema, pneumocystis pneumonia, haemorrhage, bronchoalveolar
carcinoma, ARDS, lymphangitis carcinomatosis, radiation or drug-induced pneumonitis,
hypersensitivity pneumonitis, and pulmonary veno-occlusive disease. The distinguishing
factor in our case is the history of leukaemia; this would also predispose to infection, but the
BAL has ruled pneumocystis out. In alveolar proteinosis, BAL or lung biopsy reveals intra-
alveolar deposits of proteinaceous material, dissolved cholesterol, or eosinophilic globules.
Symptomatic treatment includes whole lung lavage and multiple procedures may be required.

Pulmonary Alveolar Proteinosis (STATdx)

Causes
Autoimmune (90%)
Secondary to immunodeficiency or haematological malignancy
Congenital

Radiology
1. Ground-glass pattern
2. Crazy-paving pattern
3. Consolidation

128
Q

A 16-year-old boy presents with mild cough and dyspnoea that has persisted for several years. He is afebrile with normal WCC, but with an elevated LDH. Chest radiograph shows dense bilateral consolidation with relative sparing of the apices and costophrenic angles. No evidence of lymphadenopathy. Which is the most likely diagnosis? [Book 3 Question 9]
A. PCP pneumonia
B. Alveolar proteinosis
C. Infective exacerbation of asthma
D. Lymphoma
E. Pulmonary oedema

A

B. Alveolar proteinosis

The typical chest radiographic findings are of bilateral central and symmetric lung opacities with relative sparing of the apices. A range of findings are possible however, including asymmetric opacities and diffuse consolidation. The most common elevated serologic marker is LDH, although this is non-specific. There can be a marked disparity between the often-moderate symptoms and extensive radiologic findings.

129
Q

A 50-year-old male presents with gradual onset dyspnoea and cough. There is no preceding history. A chest radiograph shows bilateral ‘bat-wing’ consolidation, with normal heart size and no effusion. High-resolution CT shows diffuse ground-glass change with intralobular and interlobular septal thickening (‘crazypaving’ appearance). What is the most likely diagnosis? [Book 4 Question 49]
a. pulmonary oedema
b. atypical pneumonia
c. pulmonary alveolar proteinosis
d. acute respiratory distress syndrome
e. hypersensitivity pneumonitis

A

c. pulmonary alveolar proteinosis

Pulmonary alveolar proteinosis is due to accumulation of proteinaceous material in the alveoli, which causes hypoxia. There is a variable combination of airspace and interstitial changes, but diffuse ground-glass opacification and ‘crazy-paving’ are typical. A geographic distribution is also described. Lack of other features (such as cardiomegaly, lymphadenopathy, and effusion) helps to distinguish it from infection and oedema. The lack of precipitating event and slow onset makes acute respiratory distress syndrome unlikely. Hypersensitivity pneumonitis often shows no abnormality on chest radiograph, but high-resolution CT shows surprisingly marked ground-glass change and centrilobular nodules with peripheral sparing.

130
Q

A 22-year-old female patient with a known phakomatosis presents with anaemia and hypotension. CT angiogram reveals evidence of active bleeding in some of the multiple areas of low attenuation (approximately –20) seen scattered throughout both her kidneys. Which of the following features may be seen on chest CT? [Book 2 Question 42]
a. Multiple pulmonary AVMs
b. Multiple bilateral small cysts
c. Mediastinal mass
d. Thin-walled upper zone bullae
e. Cardiac rhabdomyomas

A

b. Multiple bilateral small cysts

Phacomatoses - a group of neurocutaneous syndromes consisting of
* Neurofibromatosis I and II,
* Sturge–Weber syndrome,
* von Hippel Lindau syndrome,
* Osler–Rendu–Weber syndrome and
* Tuberous sclerosis.

The above features describe multiple renal angiomyolipomas, which are a feature of tuberous sclerosis. These patients may demonstrate multiple thin-walled cysts with lower zone fibrosis (forme fruste of LAM).

131
Q

On reviewing a chest radiograph of a 26-year-old male, multiple small pulmonary nodules and a right-sided pneumothorax are identified. Which is the most likely diagnosis? [Book 3 Question 45]
A. Wegener’s granulomatosis
B. Septic emboli
C. TB
D. Alveolar cell carcinoma
E. Histiocytosis

A

E. Histiocytosis

Osteosarcoma, Wilms’ tumour and histiocytosis are causes of pulmonary nodules with pneumothorax.

132
Q

The CT chest of a 35-year-old-patient reveals parenchymal nodules, emphysema and multiple thin-walled cysts. Some of these have a bizarre configuration and are predominantly located in the upper and middle zones, sparing the costophrenic recesses and tips of the middle lobe and lingual. What is the most likely diagnosis? [Book 3 Question 14]
A. Lymphangioleiomyomatosis
B. Birt–Hogg–Dubé syndrome
C. Lymphocytic interstitial pneumonia (LIP)
D. Sarcoidosis
E. Langerhans’ cell histiocytosis (LCH)

A

E. Langerhans’ cell histiocytosis (LCH)

Early findings include interstitial infiltrate centred on the small airways, leading to the development of small parenchymal nodules. These are typically irregular, of soft-tissue density, and measure 1-5mm in diameter, although some may be over 1cm in size.

133
Q

A 30-year-old male smoker presents with a history of acute dyspnoea. CXR shows bilateral reticulo-nodular interstitial changes, predominantly in the upper and mid zones, with preservation of lung volume. There is a right-sided apical pneumothorax and a small right pleural effusion. HRCT of chest shows complex thin- and thick-walled cysts and irregular centrilobular nodules in a similar distribution with sparing of the bases. The intervening lung appears normal. What is the diagnosis? [Book 1 Question 48]
A. LAM.
B. Bronchiectasis.
C. Metastases.
D. Pulmonary Langerhans cell histiocytosis.
E. Idiopathic pulmonary fibrosis.

A

D. Pulmonary Langerhans cell histiocytosis

Pulmonary Langerhans cell histiocytosis

  • Young adult, smoker.
  • Symptoms
    o Non-productive cough, dyspnoea.
    o Less commonly fatigue, weight loss, pleuritic chest pain, fever.
  • CXR
    o Initially, small irregular nodules, bilateral, symmetric, with upper lobe
    predominance, with sparing of costophrenic angles.
    o Later stage, coarse reticular and reticulonodular pattern
    o May present with recurrent pneumothorax.
    o Pleural effusion is uncommon but may occur with pneumothorax.
  • HRCT
    o Cysts and nodules in a characteristic distribution with normal intervening lung.
    o Interstitial fibrosis and honeycombing are seen in advanced stages.
  • Treatment
    o Smoking cessation
    o Steroids
    o Chemotherapeutic agents
    o Lung transplantation.
  • Prognosis – variable.
134
Q

The HRCT of a 35-year-old patient with shortness of breath and reticulonodular disease pattern on plain chest radiograph reveals cavitating nodules with interstitial septal thickening. Which of the following diagnoses is the least likely? [Book 2 Question 44]
a. Lymphangioleiomyomatosis
b. Langerhans’ cell histiocytosis
c. Wegener’s granulomatosis
d. Sarcoidosis
e. Rheumatoid lung

A

a. Lymphangioleiomyomatosis

The cysts in LAM are thin-walled air-containing cysts, rather than cavitation in nodules.

135
Q

A 40-year-old female with known history of seizures and low IQ presents with severe shortness of breath, progressively worsening over a number of years. A chest radiograph shows extensive honeycombing throughout both lungs, which are of normal volume. A small pneumothorax is also seen on the right side. What is the most likely diagnosis? [Book 4 Question 41]
a. sarcoidosis
b. cystic fibrosis
c. tuberous sclerosis
d. lymphangiomyomatosis
e. idiopathic pulmonary fibrosis

A

c. tuberous sclerosis

The lung is involved in 5% of patients with tuberous sclerosis, with symptoms usually occurring in adult life, and it is predominantly women who are affected. There is interstitial fibrosis in the lower lungs with miliary nodular changes, which progress to honeycomb lung. Recurrent pneumothoraces occur in 50% of cases. Chylothorax may also be seen. Preservation of the lung volumes with honeycombing is seen in histiocytosis, neurofibromatosis and lymphangiomyomatosis, as well as tuberous sclerosis.

136
Q

A 38-year-old pregnant female presents with haemoptysis and shortness of breath. A chest radiograph shows a large effusion, which is drained and found to be chylous. What is the most likely diagnosis? [Book 4 Question 44]
a. histiocytosis
b. tuberous sclerosis
c. neurofibromatosis
d. lymphangiomyomatosis
e. idiopathic pulmonary fibrosis

A

d. lymphangiomyomatosis

Lymphangiomyomatosis is exclusively seen in women of child-bearing age and is worsened in pregnancy. Classically, it causes coarse reticular/reticulonodular interstitial changes, and patients develop recurrent chylous effusions and pneumothoraces. Lung volumes are preserved or may be increased (the only interstitial lung disease with increased lung volumes). It may progress to honeycombing.

Histiocytosis usually affects men, and causes cystic change in the upper lobes, small nodules and septal thickening. Tuberous sclerosis has similar features to histiocytosis but is associated with skin changes, reduced IQ, and epilepsy.

Neurofibromatosis tends to cause progressive fibrosis, but effusions and pneumothoraces are uncommon. Idiopathic pulmonary fibrosis tends to affect the older age group with a reticulonodular pattern.

137
Q

A 38-year-old male presents with a cough, not responding to antibiotics. A chest radiograph shows reticular change in both lungs with normal volumes. High-resolution CT confirms multiple thin-walled cysts with centrilobular nodules of 3–10 mm throughout the lungs. The intervening lung is normal. No pleural effusion is present. What is the most likely diagnosis? [Book 4 Question 70]
a. neurofibromatosis
b. tuberous sclerosis
c. lymphangiomyomatosis
d. histocytosis
e. usual interstitial pneumonitis

A

d. histocytosis

Honeycombing is seen with all these conditions, but the combination of normal volumes, cysts and peri-bronchial nodules is very suggestive of histiocytosis. Often the lung bases are preserved early in the disease but the whole lung is eventually involved. Lung volumes in usual interstitial pneumonitis are reduced. Neurofibromatosis causes interstitial fibrosis with little nodularity. Tuberous sclerosis causes lower zone fibrotic change with miliary nodules that progresses to honeycombing. Lymphangiomyomatosis produces coarse reticular change due to cyst formation, but nodularity is not seen. Pneumothoraces and pleural effusions are seen with neurofibromatosis, tuberous sclerosis and lymphangiomyomatosis.

138
Q

A 75-year-old man presents with worsening shortness of breath. He was a mine worker. A chest radiograph shows multiple nodules in the upper zones with a large upper-zone mass on the left. CT confirms multiple small nodules up to 5 mm with a sausage-shaped mass paralleling the mediastinum. What is the most likely diagnosis? [Book 4 Question 52]
a. coal worker’s pneumoconiosis with bronchogenic carcinoma
b. coal worker’s pneumoconiosis with progressive massive fibrosis
c. tuberculosis
d. primary lung carcinoma with metastases
e. chronic extrinsic allergic alveolitis

A

b. coal worker’s pneumoconiosis with progressive massive fibrosis

The history of mining (dust exposure) with small nodules in the upper zones is typical of pneumoconiosis. The sausage-shaped mass is characteristic of progressive massive fibrosis (PMF), although malignancy cannot be excluded. PMF is often seen to have reduced nodularity surrounding it, as it incorporates the surrounding nodules and migrates towards the hilum. Tuberculosis usually produces a generalized distribution of 2–3mm nodules in its miliary form, often with lymphadenopathy. Metastases from primary lung cancer are not uniformly small (though they can be in thyroid cancer and melanoma). Chronic extrinsic allergic alveolitis produces fibrotic changes in the mid and lower zones.

139
Q

A 65-year-old man has kept pigeons for over 20 years. He is complaining of gradually worsening shortness of breath. A CXR shows increased interstitial markings, with reduction in lung volumes. A subsequent HRCT of chest shows quite marked pulmonary fibrosis with areas of honeycomb formation. Which part of the lung is likely to be relatively spared by the fibrotic process? [Book 1, Question 12]
a. Upper zones
b. Mid zones
c. Posterior costophrenic sulci
d. Central peri-bronchovascular regions
e. Subpleural lung

A

c. Posterior costophrenic sulci

The question is leading to chronic hypersensitivity pneumonitis as the most likely underlying diagnosis. The fibrotic process, in advanced stages, affects both the subpleural lung and the peri-bronchovascular interstitium. There may be honeycomb formation at the lung bases, but unlike usual interstitial pneumonia/idiopathic pulmonary fibrosis, the honeycombing typically spares the most extreme posterior costophrenic sulci. Classically, the fibrotic process is more pronounced in the mid and upper lung zones.

140
Q

A 70-year-old man, previously working in a ship-building yard, presents with progressive breathlessness. Chest radiograph demonstrates bilateral calcified pleural plaque disease with volume loss. Lung function shows a restrictive pattern. HRCT reveals pulmonary fibrosis. The most likely site of these changes would be: [Book 2 Question 5]
a. Perihilar
b. Apical
c. Peribronchial
d. Subpleural
e. Fissural

A

d. Subpleural

Subpleural pulmonary fibrosis associated with asbestos exposure is seen mainly in a subpleural distribution towards the lung bases.

141
Q

Which of the following features distinguishes hypersensitivity pneumonitis from lymphocytic interstitial pneumonia? [Book 3 Question 50]
a. Ill-defined centrilobular nodules
b. Ground glass opacification
c. Diffuse lung involvement
d. Subpleural small nodules
e. Cystic spaces

A

e. Cystic spaces

Although ground glass changes and small centrilobular nodules are present in hypersensitivity pneumonitis, there are no cystic spaces, or thickening of the bronchovascular bundles.

142
Q

A 40-year-old non-smoking female with a month-long history of shortness of breath has an HRCT which shows patchy ground glass opacities in a centrilobular distribution, air trapping, and some spared areas of normal parenchyma. Which is the most likely diagnosis? [Book 3 Question 25]
a. Centrilobular emphysema
b. Para-septal emphysema
c. NSIP
d. Sarcoidosis
e. Hypersensitivity pneumonitis

A

e. Hypersensitivity pneumonitis

Characteristic HRCT features in hypersensitivity pneumonitis include ground glass opacity, air trapping and centrilobular ground glass opacities.

143
Q

A 40-year-old man presents with worsening breathlessness, fever and chills following a visit to an aviary earlier in the day. HRCT is most likely to demonstrate: [Book 2 Question 16]
a. Mid-zone interstitial lines
b. Areas of air-space shadowing
c. Pleural effusions
d. Lymphadenopathy
e. Crazy paving

A

b. Areas of air-space shadowing

Acute extrinsic allergic alveolitis is predominantly a type III hypersensitivity reaction. There is poor correlation between the clinical presentation and the radiological changes. Lymphadenopathy is unusual in the acute phase but is seen more commonly in recurrent disease. The early changes are mainly in the mid zones, but the fibrosis that develops in chronic extrinsic allergic alveolitis is mainly in the upper zones. In the acute phase, diffuse air-space shadowing or multiple opacities are seen.

144
Q

A 67-year-old man who was previously a manual worker presents with chest pain, which subsequently turns out to be due to myocardial ischaemia. He has a CXR performed which shows numerous small nodular densities and you suspect he has an occupational lung disease, as these densities are unchanged from previous radiographs. A subsequent HRCT of chest shows no evidence of linear interstitial change or fibrosis. Pulmonary function tests are normal. Which of the following possible causes is least likely to result in functional lung impairment?

A. Coal workers’ pneumoconiosis.
B. Silicosis.
C. Berylliosis.
D. Siderosis.
E. Asbestosis.

A

Siderosis.

  • Iron Oxide, Welders
  • Multiple small centrilobular nodules but is not usually associated with any symptoms or fibrosis.
  • If combined with silica dust it can cause silico-siderosis, which can be associated with fibrosis.

Silicosis and coal workers pneumoconiosis, secondary to inhalation of silica dust and washed coal dust, respectively, show similar features on CT. This is usually the presence of 2–5 mm nodules, mainly involving the upper and posterior lung zones. Large opacities (>1 cm) indicate progressive massive fibrosis. Calcification in lymph nodes can occur and eggshell calcification is more typical in silicosis. Berylliosis is a chronic granulomatous lung disease caused by exposure to beryllium dust or fumes. CT findings are like other granulomatous lung diseases, such as sarcoid. Fibrosis may therefore occur. Asbestosis is pulmonary fibrosis secondary to inhalation of asbestos fibers.

145
Q

A 68-year-old man presents with increasing dyspnoea. He has a history of asbestos exposure. CT of the chest demonstrates bilateral pleural thickening with calcification, and a 3 cm, rounded, lower lobe mass. This is related to an area of pleural thickening, with bronchovascular markings coursing from it towards the hilum in a curved path. What is the most likely diagnosis?

a. rounded atelectasis
b. bronchogenic carcinoma
c. tuberculosis
d. silicosis
e. round pneumonia

A

Rounded atelectasis.

Rounded atelectasis is an infolding of pleura associated with atelectasis, which occurs in the posterior lower lobes and abuts an area of pleural thickening. Often a ‘comet’s tail’ of bronchovascular markings is seen curving towards the hilum. Bronchogenic carcinoma may occur with asbestos exposure but is usually of bronchioalveolar cell type and has a latent period of 25–35 years. Round pneumonia produces spherical consolidation with air bronchograms. Silicosis may cause conglomerate masses, usually in the mid or upper zones, or progressive massive fibrosis, but reticulonodular changes predominate.

146
Q

A 67-year-old presents with shortness of breath. A chest radiograph shows a reticulonodular pattern with hilar adenopathy. Which feature on high-resolution CT would make silicosis a more likely diagnosis than sarcoidosis?

a. nodules >10 mm
b. calcified hilar lymph nodes
c. traction bronchiectasis
d. honeycombing
e. progressive massive fibrosis

A

Progressive massive fibrosis.

Silicosis is due to inhalation of silicon dioxide particles, and radiographic changes of chronic silicosis are seen 10–20 years after exposure. The nodules are typically less than 10mm, compared with sarcoid nodules, which can be more variable in size (acinar type 6–7mm, alveolar type >10mm). Calcified hilar lymph nodes (eggshell pattern), traction bronchiectasis, and honeycombing occur in both conditions and are not discriminatory. Progressive massive fibrosis is a complication of the pneumoconioses and is not seen in sarcoidosis.

147
Q

A 45-year-old man with a known atrial septal defect (ASD) presents with breathlessness and mild cyanosis. Clinical examination reveals a loud second heart sound and a prominent parasternal heave, but no signs of heart failure. Echocardiogram demonstrates a shunt reversal. Which of the following is not a usual feature on the imaging?

a. Paucity of peripheral pulmonary vasculature
b. Enlarged central PA
c. Right ventricular hypertrophy
d. Dilated pulmonary veins
e. Linear calcification of the main pulmonary arteries

A

Dilated pulmonary veins.

This patient has developed Eisenmenger syndrome with reversal of his longstanding left-to-right shunt across the ASD. All the features listed except dilated pulmonary veins are generally present.

148
Q

A patient is admitted with a comminuted femoral fracture. Initially he is quite well but goes to theatre for internal fixation of the fracture. His clinical condition deteriorates after 24 hours, and he develops fever, hypoxia, and confusion. The clinical team have noted a rash and at the same time as requesting a CT chest, request a CT brain ‘? meningitis secondary to epidural’. The CT chest reveals widespread peripheral areas of ground-glass opacification (GGO) and air-space consolidation. There are no septal lines or pleural effusions. A follow-up radiograph 10 days later reveals complete resolution. What is the most likely diagnosis?

a. Multiple pulmonary contusions
b. Pulmonary oedema secondary to anaesthetic medication
c. Fat embolism
d. ARDS
e. Pneumococcal meningitis

A

Fat embolism.

Seen in:
- Long bone fractures
- Major burns
- Pancreatitis
- Haemoglobinopathy
- Tumors
- Liposuction

Locations:
- Pulmonary
- Cerebral
- Cutaneous symptoms (petechiae secondary to coagulopathy)

The time lapse between the traumatic event and the radiographic abnormalities is usually 1–2 days, which allows differentiation from traumatic contusion. The radiographic findings resemble ARDS, although a peripheral distribution of consolidation is described. V/Q scanning will reveal multiple peripheral perfusion defects. In practice, it is the clinical features such as the rash, confusion, and coagulopathy, as well as the presence of a fracture, which raise the suspicion of fat embolism.

149
Q

A 32-year-old male presents with increasing shortness of breath following a road traffic accident, in which he sustained multiple long bone fractures. At 48 hours post-injury, his chest radiograph is normal. The next day a V/Q scan shows patchy, mottled, peripheral perfusion defects. The following day a chest radiograph shows patchy, bilateral, alveolar infiltrates. What is the most likely diagnosis? [Book 4 Question 15]

a. fat embolism
b. thrombotic embolism
c. atypical infection
d. pulmonary contusions
e. pulmonary oedema

A

Fat embolism

In a patient who has sustained multiple fractures, fat embolism should always be considered when a patient is short of breath in the presence of a normal chest radiograph. This manifests on a V/Q scan as mottled peripheral perfusion defects. Chest radiograph remains normal for up to 72 hours, when discoid atelectasis, diffuse alveolar infiltrates, and consolidation may develop. Fat embolism may precede the development of acute respiratory distress syndrome. Pulmonary contusions usually manifest earlier, within the first 24 hours.

150
Q

A young girl with known sickle cell disease presents with chest pain, fever, leucocytosis, and hypoxia. Which of the following statements is most appropriate? [Book 3 Question 40]

A. Consolidation is found more commonly than GGO on HRCT
B. Ground-glass change typically has a lobar distribution
C. The lower lobes are more frequently affected in adults
D. Infarction results in linear scarring
E. There is a correlation between severity of a chest crisis and extent of radiological findings.

A

Infarction results in linear scarring

HRCT often reveals ground-glass opacification, which does not have a lobar distribution and may have a scattered or mosaic pattern. Adults tend to have lower lobe or multilobar involvement, compared to the upper lobe pattern more often seen in children. Microvascular occlusion causes a reduction in the vascular markings and infarction results in linear scarring.

151
Q

The diagnostic role of CT in patients with pulmonary emboli is well established, but a prognostic role is being proposed as well. Which of the following has the most widely accepted prognostic value? [Book 2 Question 20]

a. PA clot burden score
b. Leftward bowing of the intraventricular septum
c. Reflux of contrast into the IVC
d. RV/LV diameter ratio
e. PA diameter measurement

A

RV/LV diameter ratio

152
Q

A previously well 42-year-old man is admitted with acute left-sided pleuritic chest pain. His SaO2 is recorded as 92%. D-Dimer assay is elevated. His mother had died suddenly at the age of 58 years. He is further investigated via CTPA, which is negative for PE. Based on his presenting symptoms, the referring consultant continues to be concerned that the patient has a PE. What advice do you offer regarding this patient’s management? [Book 1, Question 2]

A. Refer for V/Q scanning.
B. Refer for catheter pulmonary angiography.
C. Commence anticoagulation for 3 months given clinical suspicion.
D. Commence anticoagulation for 6 months given clinical suspicion.
E. No further investigation or anticoagulation required.
F. Repeat CTPA.

A

No further investigation or anticoagulation required.

  • In the setting of low probability with positive D-dimer, a good quality negative CTPA has a negative predictive value of 96%.
  • In the setting of a high pre-test probability, a negative CTPA should be followed with either venous ultrasound or MR venography (PIOPED).
153
Q

A 62-year-old man undergoes lung scintigraphy for investigation of PE. There is no prior history of PE. Which of the following scan patterns would be in keeping with a low probability for PE? [Book 1 Question 11]

A. Triple matched defect in the lower lung zone.
B. Single moderate matched V/Q defect with a normal CXR.
C. Perfusion defect with a rim of surrounding normally perfused lung.
D. No defects present on perfusion scan.
E. Four moderate segmental defects.

A

Perfusion defect with a rim of surrounding normally perfused lung.

Multiple bilateral perfusion defects with a normal ventilation scan are the classic diagnostic
findings in PE. Occluding pulmonary emboli produce segmental perfusion defects that extend
to the pleural surface. As other conditions may also produce perfusion defects, the ventilation
scan improves specificity. Non-embolic lung disease will typically have both perfusion and
ventilation abnormalities, resulting in matched defects.

V/Q scans are categorized as normal, low, intermediate, or high probability. A perfusion defect
that matches ventilation and CXR abnormalities in size and location is a triple matched defect.

A triple matched defect in the middle or upper lung zones is in keeping with low probability
but rises to intermediate probability when in the lower zones.

A single moderate matched V/Q defect, but with a normal CXR, is also of intermediate
probability.

No perfusion defect is in keeping with a normal scan and four moderate segmental defects is a
high probability scan.

A perfusion defect with a rim of surrounding normally perfused lung is known as the stripe
sign and corresponds to low probability for PE, as PE perfusion defects should extend to the
pleural surface and have no overlying stripe of perfused lung.

PIOPED II Criteria (Sn 77.4%, Sp 97.7%)

Pulmonary embolism present (high probability)
* Two or more large mismatched segmental perfusion defects or the arithmetic equivalent
of moderate and/or large defects

Nondiagnostic (low or intermediate probability)
* All other findings not falling into the pulmonary embolism present or absent categories

Pulmonary embolism absent (normal or very low probability)
* No perfusion defects
* Nonsegmental perfusion defects (e.g., pleural effusion at the costophrenic angle,
cardiomegaly, elevated hemidiaphragm, hilar enlargement, linear atelectasis), without
other perfusion defects in either lung
* Perfusion defects smaller than corresponding chest radiographic opacity
* One to three small subsegmental perfusion defects
* Two or more matched ventilation and perfusion defects with a regionally normal chest
radiograph and some areas of normal perfusion elsewhere
* Solitary triple matched defect (matched ventilation and perfusion defect with
corresponding chest radiographic opacity) in a single segment in the middle or upper
lung zone
* Stripe sign (a stripe of perfusion peripheral to a defect, best seen on tangential view)
* Large pleural effusion (occupying one-third or more of the pleural cavity), without
other perfusion defects in either lung

154
Q

In ventilation–perfusion scintigraphy, which of the following is suggestive of an intermediate probability of pulmonary embolism? [Book 4 Question 34]

a. matched non-segmental defects with a normal chest radiograph
b. multiple unmatched small perfusion defects with normal ventilation
c. large, segmental, matched defect with similar-sized opacity on chest radiograph
d. reverse mismatch
e. two large, unmatched, segmental, perfusion defects

A

Large, segmental, matched defect with similar-sized opacity on chest radiograph

With ventilation–perfusion scintigraphy, matched segmental defects are considered low probability. When there is a similar-sized area of opacification on the chest radiograph, which indicates ‘triple match,’ this becomes intermediate probability. Matched non-segmental defects, reverse mismatch, and multiple small perfusion defects are all indicators of low probability. Two large segmental perfusion defects that are not matched are considered high probability.

155
Q

A 24-year-old woman who is 28 weeks pregnant is admitted with suspected pulmonary embolism. As the on-call radiologist, her obstetrician contacts you seeking advice regarding further management. An admission CXR is normal. What investigation do you advise initially? [Book 1 Question 75]

A. Venous ultrasound.
B. Low-dose CTPA.
C. Reduced dose lung scintigraphy.
D. MRA.
E. Catheter pulmonary angiography

A

Venous ultrasound.

For pregnant patients, venous ultrasound is recommended before imaging tests with ionizing radiation are performed. Up to 29% of pregnant patients with PE will have a positive venous ultrasound, obviating the need for further imaging. The majority of the PIOPED II investigators currently recommend V/Q scanning over CTPA in the evaluation of PE in pregnant patients. The foetal dose with V/Q is similar to that with CTPA, although the effective dose per breast is much greater with CTPA. MRI requires further evaluation and gadolinium-based contrast agents have not been proven to be safe in pregnancy. The role of catheter angiography is probably limited to those patients requiring mechanical thrombectomy. It should be noted that even a combination of CXR, lung scintigraphy, CTPA, and pulmonary angiography exposes the foetus to approximately 1.5mGy of radiation, which is well below the accepted limit of 50 mGy for the induction of deterministic effects in the foetus.

156
Q

A 42-year-old female patient presents with a swollen calf, and deep venous thrombosis is suspected clinically. The D-dimer is elevated. Doppler ultrasound scan shows no thrombus in the thigh or calf veins. Spectral Doppler shows continuous signal with no respiratory variation. Which further investigation may be of value? [Book 4 Question 45]

a. no further investigation–normal findings
b. pelvic ultrasound
c. CT pulmonary angiogram
d. chest radiograph
e. echocardiogram

A

Pelvic ultrasound

Even with no clot seen, the loss of respiratory variation with continuous flow suggests a more proximal occlusion. As the limb swelling is unilateral, the most likely site of occlusion would be in the pelvic veins.

157
Q

A 56-year-old man is admitted via the accident and emergency (A&E) department. He has a past medical history of mitral valve disease. He is complaining of shortness of breath and the clinical team believe he has pulmonary oedema but ask for your opinion on his CXR to rule out infection. The presence of which of the follow features could not be attributed to cardiac failure and would make you doubt the diagnosis? [Book1 Question 10]

A. Perihilar alveolar opacities
B. Sparing of the lung periphery
C. A unilateral pleural effusion
D. Unilateral regional oligemia
E. Right upper lobe opacification

A

Unilateral regional oligemia

This represents Westermark’s sign and is associated with PE, not pulmonary oedema. The other features are consistent with cardiac failure. Focal right upper lobe oedema is associated with mitral regurgitation, where the regurgitant jet produces locally increased pressures in the right upper lobe pulmonary veins with a focal increase in oedema in that region. This can mimic consolidation on plain film but will be seen to resolve after diuresis. Pleural effusions may be unilateral in cardiac failure.

158
Q

When considering a request for inferior vena cava (IVC) filter insertion, which of the following is an absolute rather than relative indication? [Book 3 Question 10]

A. Free-floating proximal DVT
B. DVT/PE and limited cardiopulmonary reserve
C. Massive PE treated with thrombolysis/thrombectomy
D. Iliocaval DVT treated with thrombolysis/thrombectomy
E. PE with inability to achieve or maintain therapeutic anticoagulation

A

PE with inability to achieve or maintain therapeutic anticoagulation

Other absolute indications include PE/DVT with contraindication to anticoagulation, PE/DVT and complications of anticoagulation, and recurrent DVT/PE despite anticoagulation.

159
Q

A 32-year-old female presents with shortness of breath and haemoptysis. There is no leg swelling and an ECG is normal. A chest radiograph shows a triangular, pleurally based opacity in the right mid-zone with an ipsilateral effusion. Which investigation would be most helpful in making the diagnosis? [Book 4 Question 3]

a. V/Q scan
b. CT pulmonary angiogram
c. conventional pulmonary angiogram
d. high-resolution CT
e. staging CT of chest

A

**CT pulmonary angiogram **

The differential diagnosis is between pulmonary embolus or pneumonia with effusion, with investigation directed accordingly. High-resolution CT would be unhelpful due to lack of contrast. A staging chest CT is performed in the aortic phase of contrast so pulmonary arteries will be suboptimally seen. CT pulmonary angiogram is the best investigation in this case because when there is consolidation/opacification present on a chest radiograph, a V/Q scan has a high likelihood of being nondiagnostic. Conventional pulmonary angiography is a high-risk procedure and is rarely performed in modern practice.

160
Q

A 56-year-old female patient presents with shortness of breath. A chest radiograph is unremarkable. A high-resolution CT scan is performed which shows mosaic perfusion with no air trapping on expiratory scan. What is the most likely diagnosis? [Book 4 Question 4]

a. bronchiolitis obliterans
b. cystic fibrosis
c. hypersensitivity pneumonitis
d. chronic thromboembolic disease
e. asthma

A

Chronic thromboembolic disease

Mosaic perfusion is caused by abnormalities of ventilation or vascular obstruction. Expiratory scans help to distinguish causes by establishing whether there is air trapping. With no air trapping present, pulmonary emboli of any cause are most likely. Air trapping would suggest airway disease such as bronchiolitis obliterans or other causes of small airway obstruction such as bronchiectasis or cystic fibrosis.

161
Q

A 42-year-old female patient presents with dyspnoea and pleuritic chest pain. She has previously had pulmonary emboli diagnosed. A CT pulmonary angiogram is performed. Which feature would indicate chronic rather than acute thrombus on the CT? [Book 4 Question 48]

a. complete occlusion of segmental vessel
b. filling defects centrally with peripheral contrast enhancement
c. peripheral mural filling defect forming acute angle with wall
d. peripheral mural filling defect forming obtuse angle with wall
e. linear atelectasis

A

Peripheral mural filling defect forming obtuse angle with wall

Chronic emboli usually form peripheral flattened defects, forming obtuse angles with the arterial wall. Secondary changes such as hypertrophy of the right atrium and ventricle, cardiomegaly, and pulmonary hypertension may be present. Recanalization or collateral formation is suggestive of chronicity, and clot calcification also indicates chronicity.

162
Q

A 26-year-old intravenous drug user presents with reduced conscious level, associated pyrexia and malaise. Clinically, there is a systolic murmur, mild hypotension and an elevated white cell count. A chest radiograph shows multiple opacities in the mid and lower zones, some of which are cavitating. What is the most appropriate next investigation? [Book 4 Question 28]

a. CT of the thorax
b. transthoracic echocardiogram
c. white cell scan
d. MRI of the heart
e. trans-oesophageal echocardiogram

A

Trans-oesophageal echocardiogram

In this clinical scenario, the patient is most likely to have multiple septic emboli secondary to intravenous drug use. Given the multiple pulmonary abscesses and pneumonia, tricuspid endocarditis should be considered, and an echocardiogram should be performed. Trans-oesophageal echocardiogram is more sensitive to valvular vegetations and should be the investigation of choice.

163
Q

A 23-year-old female who is 23 weeks pregnant presents with pleuritic chest pain, and pulmonary embolus is suspected. She asks about the relative radiation doses for CT pulmonary angiogram and ventilation–perfusion (V/Q) scintigraphy. What is the dose for a CT pulmonary angiogram relative to a (V/Q) scan? [Book 4 Question 92]

a. CT pulmonary angiogram has a higher total body dose but a lower uterine dose
b. CT pulmonary angiogram has the same total body dose but a lower uterine dose
c. CT pulmonary angiogram has a higher total body dose and uterine dose
d. CT pulmonary angiogram has a lower total body dose and uterine dose
e. CT pulmonary angiogram has a higher total body dose but the same uterine dose

A

CT pulmonary angiogram has a higher total body dose but a lower uterine dose

The total body dose for CT pulmonary angiogram is approximately 2–3 times higher than for V/Q scanning. However, the uterine, and therefore fetal, dose is higher with V/Q scanning. Although low-dose V/Q techniques can reduce this, the uterine dose remains higher. The risk of death from pulmonary embolus outweighs any radiation risk to the patient or fetus.

164
Q

A 33-year-old driver is severely injured in a motor vehicle accident. He develops increasing dyspnoea and hypoxia and requires intubation. A chest x-ray (CXR) was normal on admission and his pulmonary capillary wedge pressure is normal. A repeat CXR performed at over 24 hours after the trauma is not normal. He is re-imaged during his intensive care unit (ICU) stay and at one point undergoes a computed tomography pulmonary angiography (CTPA), which is negative for pulmonary embolism (PE). The clinical team suspect acute respiratory distress syndrome (ARDS). Which of the following radiographic features is inconsistent with this diagnosis?

A. Bronchial dilatation on computed tomography (CT).
B. Bilateral heterogenous air-space opacities.
C. Diffuse reticular changes.
D. Pneumothorax.
E. Bilateral pleural effusions.

A

Bilateral pleural effusions.

The underlying diagnosis is ARDS.

Causes
* Direct lung injury (e.g. pneumonia, toxic gas inhalation, aspiration)
* Indirect lung injury (e.g. trauma, sepsis, pancreatitis).

Diagnosis
* PaO2 /FiO2 < 200 mmHg AND
* No evidence of left heart failure

Therefore, the presence of a pleural effusion casts doubts on the diagnosis.

Appearances
* Can mimic pulmonary oedema
* Bronchial dilatation is frequently seen on CT.
* The alveolar changes are heterogenous with a density gradient in both the cranio-caudal and antero-posterior directions.
* Radiographic changes tend to be absent for the first 24 hours (except direct lung injury), then increase to remain static for days or weeks, and then begin to resolve.
* Pneumothorax can occur secondary to ventilation.
* Reticular changes, with a predilection for non-dependent lung, may be secondary to the underlying process or to barotrauma (seen in 85% of survivors in one study; the mortality of ARDS is approximately 50%).

165
Q

In acute respiratory distress syndrome what is the first change usually seen on the chest radiograph?

a. confluent consolidation
b. pleural effusions
c. increased heart size with globular shape
d. volume loss with atelectasis
e. patchy ill-defined opacities

A

Patchy ill-defined opacities

Acute respiratory distress syndrome (ARDS) commences with interstitial oedema, progressing to congestion and extensive alveolar, and interstitial oedema and haemorrhage. The chest radiograph is often normal for the first 24 hours, before patchy opacities appear in both lungs. These progress to massive airspace consolidation over the following 24–48 hours. True volume loss, atelectasis, cardiomegaly and effusions are not seen in ARDS.

166
Q

A 37-year-old male presents to accident and emergency following smoke inhalation in a fire. He feels well and a chest radiograph is normal. The following day he re-presents feeling short of breath and unwell. What are the most likely findings on the chest radiograph now?

a. pulmonary oedema
b. pleural effusions
c. upper-zone consolidation
d. diffuse reticular change
e. pneumothorax

A

Pulmonary oedema

Inhalation of noxious gases, including smoke, produces focal or diffuse pulmonary oedema. With smoke this may be delayed by 1–2 days. Bronchiolitis obliterans may then ensue after 1–3 weeks, especially with chemical inhalation.

167
Q

A 43-year-old man with a previous history of polytrauma requiring a long period in intensive care for acute respiratory distress syndrome presents with shortness of breath. An abnormal chest radiograph prompts a high-resolution CT scan. What are the most likely findings?

a. bronchiectasis in the lower lobes
b. pleural effusions
c. fibrosis with volume loss in the upper lobes
d. reticular changes in the anterior aspects of the lungs
e. reticular changes in the posterior aspects of the lungs

A

Reticular changes in the anterior aspects of the lungs

In most patients who survive acute respiratory distress syndrome (ARDS), there is no functional deficit. Where investigated, however, the most common abnormality detected is a reticular pattern in the ventral non-dependent lung. The extent of this is negatively related to the extent of opacification in the acute phase and strongly related to the duration of mechanical ventilation. Bronchiectasis is seen in both acute and chronic phases but is less common than reticular change. Upper lobe fibrosis is not associated with ARDS.

168
Q

A 51-year-old woman has a history of a prolonged ICU admission following a subarachnoid haemorrhage 2 years previously. Despite the stormy course in ICU, she made a good neurological recovery, but has had persistent breathlessness on exertion since discharge. Her imaging shows interstitial fibrosis. Which part of the lung is likely to be relatively spared by the interstitial process?

A. Posterior aspect of the lungs.
B. Anterior aspect of the lungs.
C. Periphery of the lungs.
D. Lower zones of the lungs.
E. Mid-zones of the lungs.

A

Posterior aspect of the lungs.

The stem of the question is pointing towards ARDS during the ICU admission, resulting in pulmonary fibrosis. Classically HRCT shows relative sparing of the posterior aspect of the lungs. This pattern of sparing is unusual in other causes of peripheral fibrosis and is an important clue to the aetiology. During the acute and subacute phases of ARDS in the supine patient, the dependent portions of the lungs usually demonstrate extensive consolidation and atelectasis. It is postulated that these areas may be protected from the long-term effects of barotrauma and high oxygen exposure as they are essentially non-aerated during the acute and subacute phases.

169
Q

Which of the following features is more likely to suggest extra-lobar rather than intra-lobar sequestration?

A. Visceral pleural investment
B. Greater association with congenital abnormalities
C. Repeated infections
D. Pulmonary venous drainage
E. Presentation in adulthood

A

Greater association with congenital abnormalities

The association with congenital abnormalities is greater in extra-lobar (50%) than intra-lobar sequestration (15%). Other features of extra-lobar sequestration include systemic venous drainage, having its own pleural investment, and being symptomatic in the first 6 months of life.

170
Q

A 40-year-old man with recurrent left lower lobe pneumonias presents with fever and cough. CT shows a well-defined area of low attenuation with fine enhancing septae within the area of consolidation. An anomalous vessel supplies the mass, apparently arising directly from the aorta. What is the most likely diagnosis?

A. Intra-lobar sequestration
B. Lymphoma
C. Broncho-alveolar cell carcinoma
D. Infected bronchogenic cyst
E. Congenital cystic adenomatoid malformation

A

Intra-lobar sequestration

The presence of systemic vascular supply to the infected segment and the history of recurrent infections makes intra-lobar sequestration the most likely diagnosis.

Pulmonary Sequestration (STATdx)

  • Definition: Lung tissue not connected to tracheobronchial tree, with anolomous systemic
    arterial supply.
  • Synonyms: Rokitansky lobe
  • Pathologic Subtypes:
    o Extra-lobar sequestration
    ▪ More common in foetus
    ▪ Separate pleural covering
    ▪ Systemic venous drainage
    ▪ Histology: Dilated bronchioles, alveoli, and subpleural lymphatics
    o Intra-lobar sequestration
    ▪ Less common in foetus
    ▪ No separate pleural covering
    ▪ Pulmonary venous drainage
    ▪ Histology: Chronic inflammation and fibrosis
  • Sites:
    o 85-90% - supradiaphragmatic, 10-15% subdiaphragmatic
    o 80% left-sided, lower lung predominant
  • Imaging
    o USG
    ▪ Homogeneously echogenic, solid mass
    ▪ May be isoechoic to lung in late gestation
    ▪ Feeding vessel arising from aorta most specific imaging finding
    o Intrathoracic bronchopulmonary sequestration (BPS)
    ▪ Echogenic, triangular lesion adjacent to diaphragm
    ▪ Ipsilateral pleural effusion in 6-10%
    o Abdominal BPS
    ▪ Stomach displaced anteriorly by echogenic mass
    ▪ Separate from adrenal gland
    o MR
    ▪ Homogeneously T2 bright mass
    ▪ T2 flow void feeding vessel
    ▪ Not always necessary
  • Top Differential Diagnoses
    o Congenital pulmonary airway malformation (CPAM)
    o Hybrid lesion (BPS + CPAM)
    o Neuroblastoma
  • Clinical Issues
    o 1/3 of prenatally diagnosed lung masses
    o Associated anomalies in up to 50% - DHP, Congenital Diaphragmatic hernia
  • Treatment for hydrothorax +/- hydrops
    o Embolization
    o Resection postnatally
  • Prognosis – Excellent.
171
Q

In an investigation for lung malignancy, all the following may produce a false positive result on a PET-CT except:

a. Pulmonary hamartoma
b. Intra-lobar sequestration
c. Tuberculosis
d. Pneumonia
e. Scarring

A

Intra-lobar sequestration

Active tuberculosis, consolidation, atypical pulmonary hamartomas, and scars may cause false positive results. Uncomplicated sequestration will not demonstrate FDG uptake.

172
Q

A 30-year-old female patient with a history of recurrent lower respiratory tract infections as a child presents with cough and dyspnoea. Chest radiograph demonstrates a smaller hyperlucent left lung. Which of the following features is unlikely to be seen on HRCT?

a. Air trapping
b. Small left hemithorax
c. Diminished size of pulmonary vessels
d. Bronchiectasis
e. Left hilar enlargement

A

Left hilar enlargement

The case describes Swyer–James syndrome (Macleod syndrome). This is a post-infectious constrictive bronchiolitis which causes a small, hyperlucent lung with bronchiectasis and air trapping in expiration. The number and size of pulmonary vessels are also diminished, resulting in a small ipsilateral hilum.

173
Q

The chest radiograph of a 35-year-old male smoker, performed for immigration purposes, shows hyper-lucency of the entire right lung, with a small ipsilateral hilum. Lung markings are seen to the periphery. An expiratory film performed immediately after reveals air trapping. Which is the most likely diagnosis?

A. Hypogenetic lung syndrome
B. MacLeod syndrome
C. Unilateral proximal obstruction of the right pulmonary artery
D. Bronchial obstruction
E. Unilateral bullae

A

MacLeod syndrome

MacLeod or Swyer-James syndrome manifests as hyper-lucency with features of air trapping. The affected lung in proximal interruption of a pulmonary artery is often as opaque, or slightly opaquer, than the contralateral lung, and there is no evidence of air trapping.

174
Q

In bronchopulmonary sequestration, which of the following features would be more suggestive of intra-lobar than extra-lobar type?

a. Enclosed in visceral pleura
b. No connection to bronchial tree
c. Systemic venous drainage
d. Presentation in infancy
e. Systemic arterial supply

A

Enclosed in visceral pleura

Bronchopulmonary sequestration is a malformation consisting of a nonfunctioning lung segment with no communication to the bronchial tree and a systemic arterial supply. The intra-lobar type accounts for 75% of cases. It is enclosed in visceral pleura and presents in adulthood with pain, repeated infection, cough, and haemoptysis. The extra-lobar type is enclosed in its own pleura and presents in infancy with feeding difficulties, respiratory distress, cyanosis, and congestive heart failure. Systemic venous drainage is seen in 80% of cases of extra-lobar type, but only in 5% of cases of intra-lobar type.

175
Q

A 55-year-old female patient presents to the neurology service with features of myasthenia gravis. As part of the routine work-up a CXR is requested which demonstrates an anterior mediastinal mass. A CT scan is requested. This reveals a 5cm mass located centrally within the anterior mediastinum. This mass has poorly defined margins, resulting in obliteration of the mediastinal fat plane. There are areas of low attenuation within this lesion which have an attenuation value of 3 HU. There are stippled areas of calcification noted. There is also a right-sided pleural effusion. There is no evidence of disease elsewhere in the mediastinum, or invasion of the great vessels. You plan to carry out a CT guided biopsy, but at this stage what is the most likely diagnosis?

A. Benign thymoma.
B. Atypical thymoma.
C. Thymic carcinoma.
D. Thymic lymphoma.
E. Malignant thymic germ cell tumour.

A

Atypical thymoma

Thymoma

Whilst the ultimate differentiation between these lesions is pathological, there are several
clinical and imaging features that can help limit the differential if present. Whilst benign
thymoma, atypical thymoma, and thymic carcinoma can all present as focal mass lesions in the
thymus,

Benign thymoma would not demonstrate the locally aggressive features found on this patient’s
scan.
Atypical thymoma is a locally aggressive lesion with benign features on pathology; it has a
better prognosis than thymic carcinoma.

Atypical thymoma, thymic carcinoma, thymic lymphoma, and malignant thymic germ cell
tumours can all be locally aggressive. The presence of mediastinal lymphadenopathy, invasion
of the great vessels, or distant metastases are uncommon for atypical thymoma, but are features
of the other three tumours; none of these were present in this case.

The final key differentiating feature in this case to indicate atypical thymoma over the other
differentials is the presence of myasthenia gravis. As thymic lymphoma and malignant germ
cell tumours are not of thymic origin, they would not cause this. It is rarely a feature of thymic
carcinoma.

176
Q

A 56-year-old female is found to have a small, well-defined anterior mediastinal mass on a chest radiograph which demonstrates homogeneous soft-tissue density with some peripheral calcification on CT. On MRI it is isointense to skeletal muscle on T1WI and slightly increased signal on T2WI. It is most likely to be:

a. Thymic cyst
b. Thymoma
c. Thymolipoma
d. Thymic hyperplasia
e. Thymic carcinoma

A

Thymoma

This case describes the typical features of a thymoma. Thymic hyperplasia and thymic carcinoma are usually ill-defined abnormalities. The signal from the lesion is not typical for a thymic cyst or thymolipoma.

177
Q

With regards to imaging of primary extra-nodal lymphomas, in which of the following sites does extra-nodal Hodgkin’s disease most commonly occur?

A. Spleen
B. Thymus
C. Thyroid
D. Small bowel
E. Lung

A

Thymus

The thymus is the only extra-nodal site where Hodgkin’s lymphoma commonly occurs, and almost all cases are of the nodular sclerosing subtype.

178
Q

A 22-year-old is diagnosed with tuberculosis. Which of the following features will make a diagnosis of primary tuberculosis more likely?

a. Mediastinal enlargement
b. Septal thickening
c. Upper zone cavitation
d. Miliary nodules
e. Apical consolidation

A

Mediastinal enlargement

Mediastinal lymph node enlargement is a feature of primary tuberculosis. The other features are seen with reactivation or fibrocavitary TB. Miliary TB can be seen in any phase with haematogenous dissemination, but primary presentation is uncommon.

179
Q

Which of the following features most favours Hodgkin’s rather than non-Hodgkin’s lymphoma?

A. Posterior mediastinal nodes
B. Para-aortic lymphadenopathy
C. Mesenteric adenopathy
D. Contiguous spread
E. Hepatomegaly

A

Contiguous spread

Contiguous spread is a feature of Hodgkin’s disease, compared with non-contiguous spread in non-Hodgkin’s disease. Other differences include greater thoracic involvement and less abdominal involvement in Hodgkin’s disease.

180
Q

A 30-year-old man has a routine chest radiograph which reveals a small soft-tissue shadow resulting in loss of part of the mid-descending aortic outline. Which of the following is the most likely cause?

a. Thymoma in the left lobe of thymus
b. Hilar lymphadenopathy
c. Lingular collapse
d. Intercostal schwannoma
e. Teratoma

A

Intercostal schwannoma

The description is of a posterior mediastinal lesion obscuring part of the descending thoracic aorta. The other lesions are anterior mediastinal apart from hilar lymphadenopathy, which is hilar/middle mediastinal.

181
Q

A 4-year-old with a history of asthma is admitted with an acute exacerbation. A post-admission CXR shows evidence of pneumomediastinum. Which one of the following is a recognized sign of pneumomediastinum?

A. Spinnaker/thymic sail sign.
B. Air-crescent sign.
C. Deep sulcus sign.
D. Inverted V sign
E. Outline of the medial diaphragm inferior to the cardiac silhouette.

A

Spinnaker/thymic sail sign

Plain radiographic signs of pneumomediastinum include the Spinnaker or thymic sail sign in children, streaky lucencies in the thoracic inlet, and tubular artery sign, where air outlines the major arteries.

182
Q

A 25-year-old man has a routine chest radiograph prior to a work permit application. It demonstrates a well-defined, rounded mediastinal mass. Which of the following features on CT would make a diagnosis of bronchogenic cyst less likely? [Book 2 Question 23]

a. Soft-tissue density
b. Thick wall
c. pre-carinal location
d. Communication with tracheal lumen
e. Unilocularity

A

Thick wall

Bronchogenic cyst is the most common intrathoracic foregut duplication cyst. It could have all the above features, but in a mediastinal location, the cyst walls are usually thin. Thick-walled cysts are more likely to be oesophageal.

183
Q

A 26-year-old man suffers a blunt injury to his chest in a road traffic accident. The most common abnormality seen on CT because of blunt thoracic injury is: [Book 2 Question 13]

a. Pneumothorax
b. Pulmonary laceration
c. Haemothorax
d. Tracheo-bronchial injuries
e. Pulmonary contusion

A

Pulmonary contusion

Pulmonary contusion is the commonest manifestation of blunt trauma and indicates trauma to alveoli with alveolar haemorrhage without significant alveolar disruption. Whilst plain film changes may not be apparent for up to six hours, CT will demonstrate changes almost immediately post-trauma and signs of resolution can be seen as early as 48 hours. If unresolved, it may progress to ARDS.

184
Q

A 65-year-old male presents with a 2-month history of cough and dyspnoea and has had swelling of the face, neck, and arms for 1 week. He has had tuberculosis in the past. CT shows an irregular right paratracheal mass with calcification that is compressing the superior vena cava and right main bronchus, with patchy consolidation in the right lung. What is the most likely diagnosis? [Book 4 Question 38]

a. small-cell carcinoma
b. lymphoma
c. malignant teratoma
d. fibrosing mediastinitis
e. bronchogenic cyst

A

Fibrosing mediastinitis

Fibrosing mediastinitis is a rare condition which has two forms: focal (usually secondary to tuberculosis or histoplasmosis) or diffuse (often idiopathic). When associated with tuberculosis, it is thought to be secondary to rupture of lymph nodes in the neck or mediastinum. Calcification is seen in 63% of cases, but the mass may be difficult to differentiate from malignant conditions when calcification is not present. Symptoms and signs are due to compression/obstruction of mediastinal structures. Small-cell carcinoma and lymphoma can produce middle mediastinal masses, but calcification is rare, unless in treated lymphoma when progressive symptoms would be unlikely. Malignant teratomas are typically anterior mediastinal masses which have well-defined lobulated margins, and usually do not calcify.

185
Q

A 57-year-old man presents with chest pain and fever after an episode of vomiting. A chest radiograph shows a small left pleural effusion and pneumomediastinum. Which investigation will best establish the diagnosis? [Book 4 Question 37]

a. CT of the chest
b. barium swallow
c. water-soluble contrast swallow
d. MRI
e. transoesophageal echocardiogram

A

Water-soluble contrast swallow

Oesophageal rupture is the most likely diagnosis. CT may be able to elicit suspicious signs, such as mediastinal gas, oesophageal thickening and pleural effusion, but cannot make a definitive diagnosis. A contrast swallow is best in confirming the diagnosis, but barium should not be used due to its potential to cause a severe inflammatory reaction and worsening mediastinitis.

186
Q

A 28-year-old male is involved in a road traffic accident and sustains chest trauma. He has chest pain and bruising over the chest with reduced blood pressure. A chest radiograph shows a shift of the trachea to the right at T3–4 level with depression of the left main bronchus and loss of clarity of the aortic knuckle. Which diagnosis should be considered? [Book 4 Question 42]

a. aortic rupture
b. bronchial rupture
c. superior vena caval laceration
d. azygos vein injury
e. internal mammary artery injury

A

Aortic Rupture

Aortic injury is usually fatal, though some patients survive to reach hospital. A chest radiograph may show a variety of features including deviation of the trachea and oesophagus (position of nasogastric tube) to the right, depression of the left main bronchus, apical pleural cap and left pleural effusion. The presence of a mediastinal haematoma following trauma is more likely due to azygos or hemiazygos vein injury or possibly internal mammary or intercostal artery injury. Superior vena caval injury tends to cause right-sided mediastinal and lung changes. Bronchial rupture may be accompanied by vascular injury but would tend to present with pneumomediastinum and pneumothorax with or without collapsed lung.

187
Q

A 27-year-old female with known sickle cell disease has an outpatient appointment. She is feeling unwell, and bloods show an anaemia. A chest radiograph shows a right-sided, lobulated, para-mediastinal mass in the lower thorax with widening of the rib spaces. CT shows no calcification or bone erosion. What is the most likely diagnosis? [Book 4 Question 46]

a. neurogenic tumour
b. bronchogenic cyst
c. Bochdalek’s hernia
d. tuberculous abscess
e. extramedullary haematopoiesis

A

Extramedullary haematopoiesis

Extramedullary haematopoiesis occurs in conditions where there is prolonged anaemia. This can occur in the spleen, liver, lymph nodes, adrenals and many other sites. In the mediastinum it produces a rounded/lobulated, paraspinal soft-tissue mass, usually between T8 and T12. Unlike other causes of paraspinal masses, there is usually no pain or bone erosion. Bronchogenic cysts are of fluid density. Bochdalek’s hernia can present in adults as a paravertebral mass, which is usually asymptomatic and contains fat as well as abdominal organs (bowel, kidney or spleen). It is usually left sided. Tuberculous abscesses are usually associated with bone destruction. Neurogenic tumours are rounded/ovoid and may extend through the intervertebral foramen into the spinal canal and/or produce bone erosion.

188
Q

A 58-year-old male presents with malaise and left chest discomfort. A chest radiograph shows a well-defined mass in the left paravertebral region. CT shows that this is fatty but has soft tissue stranding within it. Some enhancement of soft-tissue elements is seen along with foci of calcification. What is the most likely diagnosis? [Book 4 Question 55]

a. lipoma
b. liposarcoma
c. hamartoma
d. neurofibroma
e. thymolipoma

A

Liposarcoma

Liposarcoma is an uncommon tumour in the thorax. It contains variable amounts of fat, with soft-tissue components that may enhance following intravenous contrast. Calcification may occur within these lesions. Lipomas are more common and occur anywhere within the mediastinum. Hamartomas usually present as soft-tissue nodules within the peripheral lung and classically show popcorn calcification. They may contain fat. Neurofibromas present as paravertebral masses, often extending into the intervertebral canal, and may have fatty attenuation due to the presence of myelin. Thymolipomas occur in the anterior mediastinum and are found in young adults.

189
Q

43-year-old female presents with stridor. A chest radiograph shows a superior mediastinal mass with narrowing of the trachea and displacement to the right. Foci of calcification are seen within it. What is the most likely diagnosis?
a. thymoma
b. teratoma
c. aneurysm of the aortic arch
d. thyroid goitre
e. lymph node mass

A

Thyroid goitre
Thyroid goitres extend into the mediastinum in 3–17% of cases. Tracheal displacement may occur due to any adjacent enlarging mass, but narrowing is specific for thyroid lesions, especially goitre. Calcification is common. Teratomas tend to arise more often in the anterior mediastinum, though they may involve the superior mediastinum by extension. Thymomas may arise in the superior or anterior mediastinum. Either of these may show calcification. Lymph node masses tend to calcify only after treatment, such as radiotherapy for lymphoma.

190
Q

In thoracic lymphoma, which feature would favour non-Hodgkin’s lymphoma over Hodgkin’s disease?
a. predominantly anterior mediastinal lymph nodal involvement
b. predominantly middle mediastinal lymph nodal involvement
c. predominantly posterior mediastinal lymph nodal involvement
d. nodal calcification
e. mass larger than 5 cm

A

Predominantly posterior mediastinal lymph node involvement
Mediastinal nodal involvement is generally more suggestive of Hodgkin’s disease, but disease is usually seen in the middle and anterior mediastinum. Posterior mediastinal involvement, with little or no anterior or middle mediastinal involvement, suggests non-Hodgkin’s lymphoma as a more likely diagnosis. Calcification can occur in either condition, nearly always post-therapy. The size of the lymph node masses is not discriminatory.

191
Q

A 20-year-old male has a chest radiograph following a slow-to-resolve chest infection. There is a mass arising from the mediastinum on the right side. Teratoma is suspected. Which feature on CT would suggest that the lesion is more likely to be benign?
a. lobulated margin
b. calcification
c. pleural effusion
d. pericardial effusion
e. mass projecting from both sides of the mediastinum

A

Calcification
A definite diagnosis as to whether a teratoma is benign cannot be made on radiological features. Features suggestive of a benign nature are a rounded lesion, projection from one side of the mediastinum, and calcification (especially if in the form of a tooth). A fat–fluid level is characteristic of a benign lesion but is rare. Features more suggestive of malignancy are lobulated margins, invasion into adjacent structures (may cause pleural or pericardial effusions), and projection from both sides of the mediastinum.

192
Q

A 30-year-old caucasian man, recently treated with bone marrow transplantation for acute myeloid leukaemia, presents with fever and cough. HRCT chest demonstrates multiple, small centrilobular nodules of soft tissue attenuation connected to linear branching opacities. What is the most likely cause of this finding?
a. Endobronchial tuberculosis
b. Primary pulmonary lymphoma
c. Invasive aspergillosis
d. Obliterative bronchiolitis
e. Diffuse panbronchiolitis

A

Endobronchial tuberculosis
The CT findings describe the ‘tree-in-bud’ pattern, which results from centrilobular bronchiolar dilatation and filling by mucus, pus, or fluid that resembles a budding tree. It is usually most pronounced in the lung periphery. All of the options provided are differentials for ‘tree-in-bud’, although infective causes are most common, classically endobronchial spread of active TB. The patient in this case is also at risk of invasive aspergillosis, although typically the ‘tree-in-bud’ pattern occurs in combination with consolidation accompanied by a halo of GGO. Obliterative bronchiolitis occurs in bone marrow transplantation in the setting of chronic graft-versus-host disease. The most sensitive CT finding in this condition is air-trapping on expiratory CT. Diffuse pan-bronchiolitis is of unknown cause but occurs almost exclusively in Eastern Asia. Primary pulmonary lymphoma is also a rare cause of ‘tree-in-bud’. Other potential differentials of this pattern include cytomegalovirus infection, cystic fibrosis, aspiration, connective tissue disease, and tumour emboli.

193
Q

A 50-year-old man has developed graft v host disease following a bone marrow transplant. He develops some breathlessness and has pulmonary function tests showing irreversible obstruction. Constrictive (obliterative) bronchiolitis is suspected. Which of the following findings on HRCT is likely to be most helpful in making this diagnosis?
a. Tree in bud opacities
b. Bronchiolectasis
c. Air-trapping
d. Centrilobular nodules
e. Cystic change

A

Air-trapping
Air-trapping is an indirect finding of small airway narrowing/obliteration and is the most common and identifying imaging feature of constrictive bronchiolitis. Air-trapping is accentuated on expiratory scans.

194
Q

A 55-year-old male patient with a history of dilated cardiomyopathy has undergone a cardiac transplantation. Now 3 months post-op, the patient presents to his cardiologists with acute lethargy, dyspnoea, and productive cough. A CXR is carried out, which shows a diffuse right-sided airspace infiltrate, with an ill-defined density noted in the right upper lobe. A CT scan is carried out, which shows patchy areas of air-space consolidation with surrounding ground-glass change in the right hemi thorax. There is an area of cavitation in the right upper lobe that has a surrounding halo of ground-glass change. The interstitial markings are not thickened. There is low attenuation noted around the heart, which has an attenuation value of –10 HU. There is a calibre change between the donor and recipient aorta. Based on the most likely pathology, as indicated by these features, what is the most appropriate first-line treatment?
a. Systemic amphotericin
b. Frusemide infusion
c. Systemic ganciclovir
d. High-dose steroid and OKT3
e. CT guided biopsy of lesion

A

Systemic amphotericin
Post-transplant complications
* Infections – most common cause of death (Aspergillus – very high mortality)
* Rejection – radiologically similar to pulmonary oedema, endomyocardial biopsy
* Accelerated atherosclerosis of the graft
* Post-transplant malignancy – Skin carcinoma, adenocarcinoma of the lung or GI tract, Kaposi sarcoma, Lymphoproliferative disorder (Leukaemia, Lymphoma)

195
Q

A 35-year-old man undergoes autologous bone marrow transplantation following successful treatment of lymphoma. Two weeks later he develops scattered bilateral progressive breathlessness and dry cough. HRCT demonstrates several areas of bilateral ground glass changes with associated reticular changes, but no effusions. What is the most likely explanation?
a. Angioinvasive aspergillosis
b. Lymphoid interstitial pneumonia
c. CMV pneumonia
d. Drug toxicity
e. Pulmonary oedema

A

Drug toxicity
Post-transplant pulmonary complications may develop in up to 40–60% of patients.
1. Neutropenia – First two weeks, most common.
2. Angio-invasive Aspergillosis – first 2-3 weeks, multiple GGO +/- cavities and peribronchiolar consolidation
3. Lyphoid Interstitial Pneumonia (LIP) – >3 months, chronic GVH
4. CMV Pneumonia – any time in the first 100 days. Multiple GGO or consolidation but not reticulation
5. Drug Toxicity – Neutropenic phase, combination of groundglass and reticular changes
6. Pulmonary oedema – Neutropenic Phase ground glass and reticular, also pleural effusion.

196
Q

Eight days after lung transplantation for alpha-1 antitrypsin deficiency, a 45-year-old man develops pyrexia, breathlessness, and desaturation. HRCT reveals perihilar heterogenous opacities and ground glass changes with new pleural effusion and septal thickening. Which of the following is the most likely cause? [Book 2 Question 45]

a. Reperfusion oedema
b. Acute rejection
c. Anastomotic dehiscence
d. Post-transplantation PCP infection may be diffuse, subpleural or peri-bronchial and may have a surrounding halo of ground-glass opacification. The findings in graft-versus-host disease are of bronchiolitis obliterans – hyperinflation, bronchial dilatation and wall thickening, reduced vascularity/mosaic perfusion, and air trapping. In the acute phase, it often presents as non-cardiogenic pulmonary oedema. Aspergillosis presents with nodular opacities or consolidation, which may have a halo of ground-glass opacification but occurs in the first 30 days. Cytomegalovirus infection usually occurs within the first 6 months after bone marrow transplantation, with a variety of appearances. Epstein–Barr virus is a causative factor in PTLD but is not in itself a cause of the pulmonary changes.

A

Acute rejection is the most likely cause as it typically presents with fever, dyspnea, and radiological changes similar to those described. Reperfusion edema typically occurs immediately post-transplantation, while anastomotic dehiscence would present differently. PCP infection usually occurs later than 8 days post-transplant.

197
Q

A 42-year-old male suffers a chest injury in a road traffic accident. The presenting chest radiograph shows fractures of the fifth and sixth ribs on the right side with patchy airspace changes. He is admitted and has supportive care. A repeat chest radiograph shows the consolidation to have largely resolved, but a rounded opacity is now present with an air–fluid level. He is otherwise well. What is the most likely diagnosis? [Book 4 Question 72]

a. abscess
b. bronchopleural fistula
c. bronchogenic cyst
d. pulmonary infarct
e. pulmonary laceration

A

Pulmonary laceration occurs following trauma disrupting the lung parenchyma. The typical appearance is of a rounded cavity containing blood and/or air. On plain films, consolidation due to contusion often obscures the laceration. The laceration appears as a rounded/ovoid opacity with a pseudocapsule of compressed lung (2–3mm), and may be fully opacified, be filled with air or have an air–fluid level. Complications are uncommon and include abscess (causes fever), bronchopleural fistula (causes pneumothorax) and progression. Bronchogenic cysts could present as an incidental finding after trauma but are usually mediastinal in location. The less common intrapulmonary bronchogenic cysts may cavitate. Pulmonary infarcts may present post-trauma due to immobilization and other risk factors causing pulmonary embolus, but these do not usually cavitate.

198
Q

A 5-year-old female presents with intermittent, colicky, epigastric discomfort with no specific features. A CT shows transverse colon protruding through a small defect anteriorly in the diaphragm in a parasternal position. What is the most likely diagnosis? [Book 4 Question 26]

a. Bochdalek’s hernia
b. Morgagni’s hernia
c. rolling hiatus hernia
d. eventration
e. septum transversum defect

A

Morgagni’s hernia is due to a developmental defect anteromedially in the diaphragm between the septum transversum and left (10%) or right (90%) costal margins. They are usually asymptomatic but can produce pain. They are usually small and present in older children or adults. A Bochdalek hernia is a posterolateral developmental defect. It tends to be large, is more common on the left and presents in infancy. Rolling hiatus hernia is herniation of the stomach through the oesophageal hiatus alongside the oesophagus. A septum transversum defect is a defect in the central tendon.

199
Q

A 52-year-old female presents with cough. She is on dialysis, but, apart from abnormal urea and creatinine, her bloods are normal. A chest radiograph is abnormal and high-resolution CT is performed. This demonstrates fluffy, nodular, 5–10 mm opacities of airspace-type appearance with foci of calcification, in an upper lobe distribution with subpleural sparing. Calcification of chest wall vessels is noted. What is the most likely cause of the appearances? [Book 4 Question 51]

a. varicella
b. chronic renal failure
c. tuberculosis
d. histoplasmosis
e. talcosis

A

Chronic renal failure is the most likely cause as the appearances are suggestive of ‘metastatic’ pulmonary nodular calcification secondary to chronic renal failure. The upper lobe distribution is due to the relative alkalinity of the upper lobes, caused by the higher ventilation-to-perfusion ratio. It is often associated with calcification of chest wall vessels. Varicella produces multiple calcified nodules of 1–3mm after the acute episode has resolved. Patients with tuberculosis or histoplasmosis, where calcified nodules are seen, have 2–5mm, well-defined nodules, and most have calcified lymphadenopathy. Talcosis characteristically produces 1mm, very-high density nodules.

200
Q

A 25-year-old male presents with dyspnoea on exertion, cough and haemoptysis. He has a history of recurrent chest infections as a child. A chest radiograph shows a hyperlucent left lung. A pulmonary embolus is suspected and a V=Q scan is arranged. This shows reduced perfusion and ventilation of the left lung, with delayed washout on ventilation. What is the most likely diagnosis? [Book 4 Question 53]

a. acute pulmonary embolus
b. Macleod’s syndrome
c. congenital lobar emphysema
d. Poland’s syndrome
e. hypogenetic lung syndrome

A

Macleod’s syndrome (Swyer–James) syndrome is a result of acute bronchiolitis in childhood causing obliterative bronchiolitis. There is usually a history of recurrent infections. Chest radiographs show hyperlucent lung with a small or normal-sized hemithorax. High-resolution CT shows reduced vascularity and attenuation of lung with air trapping. Congenital lobar emphysema presents in the first 6 months of life with respiratory distress and a hyperlucent, overexpanded lobe (usually left upper) on the chest radiograph. Poland’s syndrome is an absence of part of the pectoralis major muscle, which causes apparent hyperlucent lung, though the lung is normal. Hypogenetic lung syndrome is usually asymptomatic and causes reduced volume of lung with reduced lucency.

201
Q

A 56-year-old female has arthralgia. A chest radiograph shows erosion of the lateral ends of the clavicles, superior notching of the third to fifth ribs, and narrowing of the humeral–acromial space. What is the most likely diagnosis? [Book 4 Question 94]

a. hyperparathyroidism
b. rheumatoid arthritis
c. osteoarthritis
d. cleidocranial dysostosis
e. neurofibromatosis

A

Rheumatoid arthritis is characterized by erosion of the lateral ends of the clavicle, seen in several conditions including rheumatoid arthritis, hyperparathyroidism, and cleidocranial dysostosis. The associated rib notching and loss of the space between the humeral head and acromion (due to wasting/rupture of supraspinatus) are typically seen with rheumatoid arthritis. Clavicle erosions are not features of neurofibromatosis or osteoarthritis.

202
Q

A 59-year-old with a history of ventricular arrhythmia presents with dyspnoea on exertion. A chest radiograph shows alveolar and interstitial opacification with frank consolidation at the right lung base, which is of high density. There is no peripheral oedema or cyanosis, and the heart size is normal. What is the most likely diagnosis? [Book 4 Question 98]

a. congestive heart failure
b. sarcoidosis
c. extrinsic allergic alveolitis
d. allergic bronchopulmonary aspergillosis
e. amiodarone pulmonary disease

A

Amiodarone pulmonary disease occurs after 1–12 months of treatment. Features are of alveolar and interstitial infiltrates and high-density areas of consolidation (iodine attenuation). Heart failure usually gives a ‘bat-wing’ distribution of consolidation with increased heart size and vascular congestion, and often cyanosis. Sarcoidosis may produce reticulonodular changes, which coalesce to form areas of consolidation. Acute extr